Final exam review LPN to RN

¡Supera tus tareas y exámenes ahora con Quizwiz!

A client made a formal request to review his or her medical records. With review, the client believes there are errors within the medical record. What is the most appropriate nursing response?

"According to HIPAA legislation, you have a right to request changes to inaccurate information."

A nurse manager is discussing a nurse's social media post about an interesting client situation. The nurse states, "I didn't violate client privacy because I didn't use the client's name." What response by the nurse manager is most appropriate?

"Any information that can identify a person is considered a breach of client privacy."

A pediatric client's caregiver states, "I will never give my child vaccinations." What is the priority nursing response?

"Help me understand your thoughts about vaccinations."

Which nurse-to-provider interaction correctly utilizes the SBAR format for improved communication?

"I am calling about Mr. Jones. He has new onset diabetes mellitus. His blood glucose is 250 mg/dL (13.875 mmol/L), and I wondered if you would like to adjust the sliding scale insulin."

The parents of a hospitalized 10-year-old ask the nurse if they can review the health care records of their child. What is the appropriate response from the nurse?

"I will arrange access for you to review the record after you put your request in writing."

Which actions should the nurse perform to limit casual access to the identity of clients? Select all that apply.

- Obscuring identifiable names of clients and private information about clients on clipboards - Placing fax machines, filing cabinets, and medical records in areas that are off-limits to the public - Keeping record of people who have access to clients' records

The nurse is teaching an unlicensed assistive personnel (UAP) about fire safety. Which UAP statement demonstrates that teaching has been effective? "I will rescue clients from harm before doing anything else." "I will sound the alarm before I start moving a patient from a room." "I will leave all doors open after rescuing patients." "I know that nurses are the only ones who can extinguish a fire."

"I will rescue clients from harm before doing anything else." Rationale: Remember, RACE = Rescue, alarm, confine, and extinguish.

A nurse is requesting to receive the change-of-shift report at the bedside of each client. The nurse giving the report asks about the purpose of giving it at the bedside. Which response by the nurse receiving the report is most appropriate?

"It will allow for us to see the client and possibly increase client participation in care."

what are the characteristics of a magnet facility?

- decentralized-decision making - self-governance at the unit level - respect and acknowledgement of autonomous practice

The nurse is caring for a client whose spouse wishes to see the electronic health record. What is the appropriate nursing response?

"Only authorized persons are allowed to access client records."

The nursing student is discussing the need for a care plan with the instructor. What is the mostappropriate explanation by the instructor for nursing care plan development?

"The care plan is required for every client by The Joint Commission."

A nurse is providing a change-of-shift report on a client who has had a restless night, is experiencing anxiety, and requires frequent repositioning. Which statement indicates a correct way of conducting an effective handoff at change of shift?

"The client had a good deal of anxiety last night and requested to be turned and repositioned frequently."

Which statement indicates that a family understands the teaching that has been provided by the nurse related to car seat safety for their 3-year-old child? "We place our child in a front-facing car seat in the back seat of the car." "We place our child in a front-facing car seat in the front of the car." "We place our child in a rear-facing car seat in the back seat of the car." "We place our child in a rear-facing car seat in the front of the car."

"We place our child in a front-facing car seat in the back seat of the car." Rationale: Children over age of 2 should be placed in FRONT-FACING car seat based on weight and height. Less than 2 should be BACK-FACING car seat.

Before the RN delegates any nursing intervention they must consider 5 factors

(1) stability of the patient's condition (2) complexity of the activity (3) potential for harm (4) predictability of the outcome (5) other patient needs

A nurse is giving a verbal report to a health care provider using the ISBAR communication technique. The client being discussed has signs and symptoms of fluid volume deficit. Which statements should the nurse include in the report? Select all that apply.

- "I am the nurse assigned to the client." - "The client reports dizziness when walking." - "The client vomited twice and has dry mucous membranes." - "Current blood pressure is 90/50 mm Hg with a pulse of 112 bpm." - "All of the orders have been completed."

Which are appropriate actions for protecting clients' identities? Select all that apply.

- Document all personnel who have accessed a client's record. - Place light boxes for examining X-rays with the client's name in private areas. - Have conversations about clients in private places where they cannot be overheard.

Which are uses of documentation in client records? Select all that apply.

- Quality improvement - Research - Decision analysis - Financial reimbursement

A nurse is documenting care for an older adult client who is recovering from a mild stroke. Which documentation entries follow the recommended guidelines for communicating and documenting client information? Select all that apply.

- The client rates pain as 2 compared to a 7 yesterday. - Vital signs returned to normal. - Radial pulse 72, strong and regular

With input from the staff, the nurse manager has determined that bedside reporting will begin for all client handoff at shift change to improve client safety and quality. When performing bedside reporting, what information should the nurse include? Select all that apply.

- any abnormal occurrences with the client during the shift - identifying demographics, including diagnosis - current orders

what do the 14 characteristics, Forces of Magnetism identify?

- quality patient care - excellent nursing care - innovations in nursing

Lab data indicating infection are?

-Elevated WBC count: normal is 5000 to 10,000/mm3 -Increase in specific types of WBC -Elevated erythrocyte sedimentation rate (RBCs settle more rapidly to the bottom of a tube of whole blood when an inflammation is present) -Presence of pathogen in urine, blood, sputum, or draining cultures

A nurse graduate applies for a job working in a hospital that achieved Magnet® status. Which conditions would this nurse expect? Select all that apply.

-Focus on positive client care outcomes -Autonomous, accountable professional nursing practice -Supportive nurse managers

The nurse is taking verbal medication prescriptions from the provider by hand to be documented in the clients eMAR for administration of medication. How should the nurse correctly document this information?

0800-Amoxicillin 250mg PO with water. J. Doe, RN.

A nurse has administered 1 unit of glucose to the client as per order. What is the correct documentation of this information?

1 Unit of glucose

Owen is a 15-year-old client who is waking up postoperatively. He became combative and tried to strangle one of the nurses. A support team was called and 4-point restraints were applied in this emergent situation. How soon does a licensed provider need to assess the client and place the restraint order? 15 minutes 4 hours 1 hour 30 minutes

1 hour Rationale: Restraints can be placed immediately for emergency situations without an order, but patients need to be assessed within 1 hour of restraint placement (apparently)

A nurse is applying restraints to a confused client who has threatened the safety of a roommate. Which actions would the nurse perform when properly applying restraints to a client? Select all that apply. Check agency policy for the application of restraints and secure a physician's order. Choose the most restrictive type of device that allows the least amount of mobility. Pad bony prominences. For a restraint applied to an extremity, ensure that the restraint is tight enough that a finger cannot be inserted between the restraint and the client's wrist or ankle. Fasten the restraint to the side rail. Remove the restraint at least every 2 hours or according to agency policy and client need.

1. Check agency policy for the application of restraints and secure a physician's order. 2. Pad bony prominences. 3. Remove the restraint at least every 2 hours or according to agency policy and client need. Rationale: -You need MD orders for restraints after having a justifiable reason such as threatening the safety of another pt, pad bony prominence to avoid injury to those areas so circulation is not impaired -Least restrictive type -A finger should be able to find so that the restraint is not too tight -Fastened to non-moving parts of the bed = side rails can injure the attached body parts when someone moves it

A client has been diagnosed with a glioblastoma and the care team has determined that this brain tumor is inoperable. Which aspects of the the client's subsequent care demonstrate adherence to the Quality and Safety Education for Nurses (QSEN) competencies? Select all that apply. The care team meets with the client and family promptly to identify their preferences for treatment. The care team balances the best available evidence about glioblastoma treatment with the client's preferences. Nurses proactively identify threats to the client's safety that may occur as treatment is provided. Each member of the care team uses the best available technology to organize and provide care. Treatments are chosen with the goal of minimizing the financial burden on the health care institution.

1. The care team meets with the client and family promptly to identify their preferences for treatment. 2. The care team balances the best available evidence about glioblastoma treatment with the client's preferences. 3. Nurses proactively identify threats to the client's safety that may occur as treatment is provided. 4. Each member of the care team uses the best available technology to organize and provide care. Rationale: 1. is patient centered care, 2. use of knowledge and evidence and team collab, 3. Safety, 4. team collab and use of technology (informatics) -QSEN is NOT concerned with financial probs

The nurse observes an unlicensed assistive personnel (UAP) collecting a urine specimen from a client with Staphylococcus aureus infection. Which action by the UAP would require the nurse to intervene?

removes gloves and walks out of the room

During discharge planning, the nurse is assessing home safety for a client who has repeatedly fallen. Which condition increases the client's risk for falls? Select all that apply. climbs two flights of stairway to get to his bedroom prefers to use the bathtub when taking a bath drinks 2 shots of alcoholic beverages before dinner takes a diuretic pill early in the morning uses non-skid socks all day

1. climbs two flights of stairway to get to his bedroom 2.prefers to use the bathtub when taking a bath 3. drinks 2 shots of alcoholic beverages before dinner 4. takes a diuretic pill early in the morning Rationale: Uh the non-skid socks mean they aren't going to slip (LOL crey), these are all safety risks. Alcohol is too because it can impair judgement = lead to falls

what is are the 5 roles of management? *POSDC*

1. plan 2. organizing 3. staffing 4. directing 5. controlling

what are the new 5 model components of a magnet facility?

1.Transformational leadership 2.Structural empowerment 3.Exemplary professional practice 4.New knowledge, innovation, and improvements 5.Empirical quality results

when the manager is hiring, orienting, scheduling to facilitate team building they are fulfilling which role?

staffing

The new nurse is having difficulty managing the time required to care for a group of complex clients and is several hours behind in completing nursing interventions. Which intervention should the nurse complete first?

Administer a dose of digoxin that is two hours behind schedule.

The nurse is planning care for a patient who requires bilateral arm restraints. Which information is important to understand when planning care for this patient? 1. Their use adequately prevents injuries 2. They require a physician's order to be applied 3. Reasons for their use must be clearly documented 4. Most patients recognize that they contribute to their safety

3. Reasons for their use must be clearly documented Rationale: Restraints can be applied in emergency situations to protect patients from harming themselves or others. Physican's order must be obtained within 12hr.

A nurse is assessing the central venous pressure of a client who has a fluid imbalance. Which reading would the nurse interpret as suggesting an ECF volume deficit?

3.5 cm H2O normal pressure is approximately 4 to 11 cm H2O.

how long is magnet status retained for?

4 years, then the hospital reapplies

An appropriately worded goal associated with the nursing diagnosis Risk for Injury is, "The patient will be: 1. Taught how to call for help to ambulate." 2. Kept on bed rest when dizzy." 3. Restrained when agitated." 4. Free from trauma."

4. Free from trauma." Rationale: The first 3 are planned interventions, while the last one is a goal

The nurse is reviewing the chart of a client receiving oxygen therapy. The nurse would question which supplemental oxygen prescription if written by the health care practitioner? 10 L/min oxygen via Venturi mask 8 L/min oxygen via partial rebreather mask 8 L/min oxygen via nasal cannula 12 L/min oxygen via nonrebreather mask

8 L/min oxygen via nasal cannula Explanation: The correct amount delivered FiO2 for a nonrebreather mask is 12 L/min; 8-11 L/min for partial rebreather mask; 4-10 L/min for Venturi mask; and 1-6 L/min for nasal cannula. However, per nasal cannula it may be no more than 2-3 L/min to for a client with chronic lung disease.

The nursing student who is learning skills during campus lab identifies which of the following statements about bedpans to be true? a) The rounded shelf of a regular bedpan should be placed under the client's buttocks. b) A regular bedpan is generally more comfortable for clients than a fracture bedpan. c) A fracture pan is preferred for urination and a regular bedpan is preferred for defecation. d) A fracture bedpan should used only for clients who have fractures of the femur or lower spine.

A

delegation

transfer of responsibility while retaining accountability for the outcome

this type of leadership pushed a hospital to achieve magnet status

transformational leadership

Which statement describes the person who is likely the most motivated to learn?

A 70-year-old female who is the client's spouse and is learning the care so the client can come home

Which clinical situation is addressed by the provisions of the Health Insurance Portability and Accountability Act (HIPAA)?

A client has asked a nurse if he can read the documentation that his physician wrote in his chart

A nurse is performing a physical assessment of a client's urinary system. Which nursing actions are appropriate during this assessment? Select all that apply. a) If using a bedside scanner, the nurse places the client in a supine position. b) The nurse measures the height of the edge of the bladder below the symphysis pubis. c)The nurse inspects the urethral orifice for any signs of inflammation, discharge, or foul odor. d) The nurse places male clients in the dorsal recumbent position for good visualization of the meatus. e) The nurse retracts the foreskin of an uncircumcised male client to visualize the meatus. f) The nurse assess the client's urine for color, odor, clarity, and the presence of any sediment.

A, C, E, F

Nurses use social media to share ideas, develop professional connections, access educational offerings and forums, receive support, and investigate evidence-based practices. Which is an example of the proper use of social media by a nurse?

A nurse uses a disclaimer to verify that any views expressed on Facebook are his or hers alone and not the employer's.

A nurse is caring for an 18 month-old boy status post a tracheostomy. He is recovering well and wanting to be more active. The nurse selects a toy from the playroom for him to play with. Which toy is most developmentally appropriate? Marbles Dominos A rocking horse A beaded bracelet

A rocking horse Rationale: Obvi he can choke on the other ones. Rocking horses are a great toy for development of leg muscles.

The nurse is preparing to measure the depth of a client's tunneled wound. Which of the following implements should the nurse use to measure the depth accurately?

A sterible flexible applicator moistened with saline

The nurse is assessing the wounds of patients in a burn unit. Which wound would most likely heal by primary intention?

A surgical incision with sutured approximated edges.

Which client will have more adipose tissue and less fluid?

A woman

The nurse has entered a client's room to empty the client's urine collection bag at the end of a busy shift. The nurse realizes that the client's urine output is 75 mL over the past 8 hours. The nurse would recognize that the client is experiencing: A) oliguria B) anuria C) nocturia D) polyuria

A) Oliguria

A nurse assessing an elderly patient finds that the patient has had four urinary tract infections in the past year. Which physiologic change of aging would the nurse suspect is the cause? a) Decreased bladder contractility b)Diminished ability to concentrate urine c)Decreased bladder muscle tone d) Neurologic weakness

A) decreased bladder contratcility

what are the 3 stages of change?

unfreezing: change is recognized moving: change is initiated refreezing: change is operational

A client with a diagnosis of colon cancer has opted for a treatment plan that will include several rounds of chemotherapy. What vascular access device is most likely to meet this clients needs?

AN implanted central venous access device

A client accuses a nurse of negligence when he trips when ambulating for the first time since hip replacement surgery. Which action is the best defense against allegations of negligence?

Accurately documenting client care on the client record

Which age group is at risk for fluid and electrolyte imbalances resulting from fad dieting?

Adolescents

Then nurse would recognize which of the following clients as being particularly suseptable to impaired wound healing

An obese woman with a history of type 1 diabetes

The nurse has just admitted a client with a latex allergy to the medical-surgical nursing floor. Which is the priority nursing intervention? Flag the room door. Apply an allergy-alert identification bracelet on the client. Notify the interdisciplinary healthcare team to use nonlatex equipment. Teach client to wear Medic-Alert bracelet.

Apply an allergy-alert identification bracelet on the client. Rationale: Apply this bracelet so ANY member of the interdisciplinary team can quickly identify the latex allergy.

HAND HYGIENE: USING AN ALCOHOL-BASED HANDRUB

Apply product to the palm of one hand, using the amount of product recommended on the package (it will vary according to the manufacturer but usually is 1 to 3 mL). Rub hands together, making sure to cover all surfaces of the hands, fingers, and in between the fingers. Also, clean the fingertips and the area beneath the fingernails. Continue rubbing until the hands are dry (at least 15 seconds).

Rich source of potassium?

Apricots

The nurse is caring for a 70-year-old client with a fractured wrist. Which is the best method to determine whether the client has retained the information taught?

Ask the client to recall after approximately 15 minutes.

A school-age child is admitted to the emergency room with the diagnosis of a concussion following a collision when playing football. After the collision, the parents state that he was "knocked out" for a few minutes before recognizing his surroundings. What is the priority assessment when the nurse first sees the client? Assessment of head circumference Assessment of vital signs and respiratory status Evaluation of all of his cranial nerves Initiation of a peripheral intravenous (IV) line for fluid administration

Assessment of vital signs and respiratory status. Remember PRIORITY or IMMEDIATE includes ABCs. Assessment of vs and respiratory status is a priority for this client. Eval of his cranial nerves should NOT take priority over cardiopulmonary assessment. Assessment comes before intervention in the nursing process.

A nurse is in charge of a large group of employees on a busy surgical floor. Today's care must be completed early due to a special event involving most of the employees. Which management style would work best in this situation?

Authoritarian

A nurse manager informs the staff members during a meeting that unlicensed assistive personnel will no longer be allowed to check clients' blood glucose levels. The nurse manager informs the group that this is a new policy on the unit and that discussions will not change the enforcement of this policy. What type of leadership style is the nurse manager demonstrating?

Autocratic

On a medical unit, the nurses complain that they have no voice in the decisions that are made in the operation of the unit. The nurses state they are always told by the nurse manager to perform tasks instead of being asked. Which of these best describes the leadership style of the nurse manager?

Autocratic

Quantum leadership is effective for which scenarios?

unpredictable events and changing environments present themselves

A home care nurse visits a client diagnosed with depression who informs the nurse that he has been prescribed amitriptyline. What would the nurse include when educating the client about the effects of this medication? a) It causes urinary retention. b) It causes urine to turn blue-green. c) It decreases sensation of bladder fullness. d) It decreases glomerular filtrate rate.

B) it causes urine to turn blue-green

When planning care for a client with a Foley catheter, which actions should the nurse include? (Select all that apply.) a) Use powder or lotion in the perineal area. b) Encourage fluid intake, unless contraindicated. c) Record volume and character of the urine. d) Maintain a closed urinary catheter system. e) Change the indwelling catheter regularly.

B, C, D

The nurse is working at a facility that is applying for Magnet® Recognition. The nurse knows that compared with other hospitals, Magnet® hospitals have which direct effect on client care?

Better patient outcomes

The nurse is preparing to don a gown to care for a client requiring contact precautions. When should the nurse don the gown?

Before entering the client's room

A nurse is caring for a client who has a pressure ulcer on the left great toe. The client is scheduled for debridement the next morning. Based on the red-yellow-blue (RYB) Wound Classification System, which of the following classifications should the nurse document?

Black Classification

The nurse is teaching the caregiver of a school-age child about safety. Which teaching will the nurse include? Supervise your child on the changing table. Place all household cleaners out of reach. Buy protective sporting equipment. Peer pressure causes children of this age to task risks.

Buy protective sporting equipment. Rationale: Children at this age group are learning how to play sports and being more active. This is most appropriate. 1. They don't need to be changed (infants) 2. They probably won't chug any household cleaners because they have enough common sense not to. Common for toddlers 4. Peer pressure doesn't really start until like teens

Before starting the education process, the nurse should determine the preferred learning style, age and developmental level, capacity to learn, motivation level, readiness to learn, and learning needs of the client. How does this help the nurse in the client's health education?

By implementing effective teaching

A nurse assesses the urine of a patient who is using a bedpan and finds that it is a dark brown color. What medication might be causing this effect? a) Phenazopyridine b) Amitriptyline c) Levodopa d) Diuretics

C) Levodopa

A 75-year-old man was admitted to the hospital for altered mental status. He had been in his usual state of good health until this morning when a nurse at the long-term care facility where he lives noticed that he was confused. Shortly after being admitted to the hospital, he became combative and had to be restrained. His bed linens have to be changed frequently because of urinary incontinence. Which nursing diagnosis best describes this client's condition? A) Stress incontinence B) Urge Urinary incontinence C) Functional incontinence D) Total Urinary incontinence

C) functional incontinence

four categories that are responsible for a majority of HAIs in the acute care hospital setting

Catheter-associated urinary tract infection (CAUTI) Surgical site infection (SSI) Central line-associated bloodstream infection (CLABSI) Ventilator-associated pneumonia (VAP)

The nurse manager is reviewing the QSEN quality and safety competencies for nurses. Which competencies are included in this initiative? Select all that apply. Client-centered care Teamwork and collaboration Establishment of clinical career ladders Revamping the licensing requirements for foreign-educated nurses Quality improvement (QI)

Client-centered care, Teamwork and collaboration, and Quality improvement (QI) Rationale: The Quality and Safety Education for Nurses (QSEN) project is designed to provide a framework for knowledge, attitudes, and skills needed for future nurses. 6 competencies = 1. Client-centered care, 2. teamwork/collab, 3. evidence-based practice, 4. quality improvement, 5. safety, 6. informatics.

A nurse is cleaning the wound of a gunshot victim. Which of the following is recommended guideline for this procedure?

Clean the wound from top to bottom and center to outside.

When interacting with a patient, the nurse answers, "I am sure everything will be fine. You have nothing to worry about." This is an example of what type of inappropriate communication technique?

Cliché

A nurse is preparing to document client care in the electronic medical record using the SOAP format. The client had abdominal surgery 2 days ago. How would the nurse document the "S" information?

Client states, "I have more pain in my belly today than I did yesterday. My pain is about a 7 out of 10."

A nurse accidentally gives a double dose of blood pressure medication. After ensuring the safety of the client, the nurse would record the error in which documents?

Client's record and occurrence report

Verification for blood transfusion?

Clients name ID number Unit number ABO group

When caring for a client, the nurse observes that the client enjoys reading books and magazines. In which learning domain does the client's learning style fall?

Cognitive

Which of the following actions should the nurse perform when applying negative pressure wound therapy?

Cut foam to the shape of the wound and place it in the wound.

A staff nurse works on a medical unit where staff retention is very high. There is a sense of equality between the leader and the staff nurses. The unit decisions and activities are shared between the leader and the group. The designated nurse leader practices which leadership style?

Democratic

As professionals, nurses generally respond well to this style of leadership?

Democratic leadership

Which type of incontinence is caused by pelvic floor muscle weakness? A) urge B) overflow C) functional D) stress

D) stress

The nurse is caring for a client with a Foley catheter in place who has a prescription for a sterile urine specimen for culture and sensitivity. The nurse implements which techniques to obtain the prescribed urine specimen? (Select all that apply.) a) The nurse allows the urine to flow from the collection bag into the specimen container. b) The nurse disconnects the catheter and allows the urine to drip into the specimen container. c) The nurse clamps the tube below the access port for 40 minutes to allow urine to accumulate. d) The nurse uses a syringe to withdraw urine from the port. e)The nurse dons clean gloves and cleanses the port with aseptic solution.

D, E

A nurse manager of a hospital unit is working within a decentralized management structure. Which nursing action best exemplifies this type of system?

Decisions are made by those who are most knowledgeable about the issue.

A med surg nurse is assessing woulds of patients. Which wound coplications accurately described below?

Dehiscience- patrtial or total disruption of wound layers Evisertation Postoperative fistulas

The wound care nurse evaluates a client's wound after being consulted. The client's wound healing has been slow. Upon assessment of the wound, the wound care nurse informs the medical-surgical nurse that the wound healing is being delayed due to client's state of dehydration and dehydrated tissues in the wound that are crusty. What is another term for localized dehydration in a wound?

Desiccation Desiccation is localized wound dehydration. Maceration is localized wound overhydration or excessive moisture. Necrosis is death of tissue in the wound. Evisceration is complete separation of the wound, with protrusion of viscera through the incisional area

5. A nurse working in a busy emergency department is caring for a teenage patient who presents with a burning pain in his mouth, edema of the lips, vomiting, and hemoptysis. The teen admits that he was playing a dare game with friends and was forced to swallow a drain opener preparation. What would be the nurse's priority intervention? Induce vomiting and call the primary care provider. Perform stomach lavage and call the poison control center. Give activated charcoal orally and call the physician. Dilute the poison with milk and call the primary care provider.

Dilute the poison with milk and call the primary care provider. Rationale: Drain opener = dilute with milk or water. For vitamin preps, stomach lavage (stomach pumping = cleaning out the contents of the stomach) = to remove undigested pills Acetaminophen poisoning, activated charcoal may be used.

A nurse informs the client that the client has no choice and must take a bath in the morning. What type of leadership does this exemplify?

Directive leadership

The nurse is removing soiled gloves after assisting with a sterile procedure. Which actions follow recommended guidelines for this procedure? Select all that apply.

Discard the gloves in appropriate container, removing additional PPE, if used, and performing hand hygiene. Use the dominant hand to grasp the opposite glove near cuff end on the outside exposed area. Remove the glove by pulling it off, inverting it as it is pulled, and keeping the contaminated area on the inside.

According to the Candian Nurses Association (CNA), what is the primary source of evidence to measure performance outcomes against standards of care?

Documentation

The nurse is changing a dressing of a patient with a gunshot wound. What nursing action would the nurse provide?

Dressing that absorbs exudate, but maintains moist environment.

The nurse is caring for a client with "hyperkalemia related to decreased renal excretion secondary to potassium-conserving diuretic therapy." What is an appropriate expected outcome?

ECG will show no cardiac dysrhythmias within 48 hours after removing salt substitutes, coffee, tea, and other K+-rich foods from diet. Supplemental potassium should not be added to the client's intake

A nurse is assessing patient wounds would document which examples as healing normally?

Edges h ealing, with crust wound that doesnt feel hot forms excudate

A nursing student is nervous and concerned about working at a clinical facility: What is the BEST decrease anxiety and ensure success in the student's provision of care?

Engaging in self-talk to plan the day and decrease fear.

In a helping relationship, the nurse would most likely perform what action?

Establish communication that is continuous and reciprocal.

An 18-year-old client is being treated for a sexually transmitted infection. The parent of the client comes to the clinic demanding information regarding the care provided since the child is covered on the parent's insurance. Which response by the nurse is most appropriate?

Explain the reason why information cannot be disclosed.

When a nurse picks up a client's contaminated tissue without gloves and fails to wash the hands sufficiently, the nurse provides for the client's organisms to be spread by which type of transmission?

contact

During a nursing staff meeting, the nurses resolve a problem of delayed documentation by agreeing unanimously that they will make sure all vital signs are reported and charted within 15 minutes following assessment. This is an example of which characteristics of effective communication?

Group decision making. Group identity. Group patterns of interaction. Group cohesiveness.

The nurse is helping a confused client with a large leg wound order dinner. Which is the most appropriate food for the nurse select to promote wound healing?

Fish- Fish is high in protein to promote protein

The nurse's morning assessment of a client who has a history of heart failure reveals the presence of 2+ pitting edema in the client's ankles and feet bilaterally. This assessment finding is suggestive of:

Fluid volume excess

A nurse is measuring intake and output for a patient who has congestive heart failure. What does not need to be recorded?

Fruit consumption

What is the most effective way to prevent the spread of infectious disease?

Hand hygiene

Examples of medical asepsis(clean technique)?

Hand hygiene using clean gloves cleaning the environment routinely

An experienced nurse is teaching a student nurse about the proper use of hand hygiene. Which guideline should the nurse provide to the student?

Hand hygiene is needed after contact with objects near the client.

when the manager is implementing mechanisms for ongoing evaluation they are fulfilling which role?

controlling

Which statement should the nurse include in the education plan regarding safety issues for a group of adult clients? In most age groups, motor vehicle accidents are major causes of death. Suicide is the leading cause of death in adults and adolescents. Occupational safety practices can eliminate all workplace hazards. Environmental lead exposure is a primary cause of death in adult clients.

In most age groups, motor vehicle accidents are major causes of death. Rationale: MVA continues to be a MAJOR cause of death for all ages groups. Also, remember NOT to choose answers that have "always", "never", "all" in them. Safety practice can reduce, but not COMPLETELY ELIMINATE, workplace risks.

A nurse in a rehabilitation division state to the head nurse: "I need a day off and you didn't give it me!" The head nurse replies, "Well, I wasn't aware you needed the day off, and it wasn't possible since staffing is so inadequate." Instead of this exchange what communication by the nurse have been more effecitive?

I would like to discuss my schedule with you. I requested the 8th of August off for a doctor's appointment. Could I make an appointment?

13. An older resident who is disoriented likes to wander the halls of his long-term care facility. Which action would be most appropriate for the nurse to use as an alternative to restraints? Sitting him in a geriatric chair near the nurses' station Using the sheets to secure him snugly in his bed Keeping the bed in the high position Identifying his door with his picture and a balloon

Identifying his door with his picture and a balloon Rationale: This method allows patient to wander, but he is able to find his room easier. The next alternate is not letting the patient wander. Geriatric chair and sheets are forms of physical restraint. Leaving bed in high position leaves the patient with the risk of falling.

The nurse is documenting a variance that has occurred during the shift. This report will be used for quality improvement to identify high-risk patterns and, potentially, to initiate in-service programs. This is an example of which type of report?

Incident report

The nurse working with the hospital's infection control team is attempting to decrease the transmission of healthcare-associated pathogens. Which intervention will be most effective?

Incentivizing health care workers to utilize hand hygiene

Many chronic medical problems adversely affect a person's ability to maintain normal fluid, electrolyte, and acid-base homeostasis. What describes complications related to liver disease?

Increased plasma levels of antidiuretic hormone lead to water excess. In addition to increased plasma levels of antidiuretic hormones, plasma levels of albumin decrease, so that the distribution of extracellular fluid changes, vascular volume decreases, and interstitial volume increases. Complications often lead to ascites

The health care provider is in a hurry to leave the unit and tells the nurse to give morphine 2 mg IV every 4 hours as needed for pain. What action by the nurse is appropriate?

Inform the health care provider that a written order is needed.

A nurse is providing instruction to a patient regarding the procedure to change a colostomy bag. During the teaching session, the patient asks, "What type of foods should I avoid to prevent gas? The patient's question allows for what type of communication on the nurse's part?

Information clarification.

A nurse on a cardiac care unit oversees the care of diverse clients' cardiac health problems. Which action can be most appropriately delegated to a licensed practical nurse (LPN)? Application of a client's cardiac monitor Initiation of manual external defibrillation Initiation of CPR for a client who is found unresponsive Collecting an arterial blood sample

Initiation of CPR for a client who is found unresponsive Explanation: The initiation and provision of cardiopulmonary resuscitation is appropriate for all health care providers. Depending on the state's nurse practice act and the organization's policies and procedures, an LPN may or may not be able to perform the other listed actions.

Nurses at a health care facility maintain client records using a method of documentation known as charting by exception (CBE). What is a benefit of this method of documentation?

It provides quick access to abnormal findings.

A nurse leader on a unit allows the staff to make all decisions and direct themselves, including filling out the work schedule. The nurse leader is practicing which leadership style?

Laissez-faire

The nurse manager calls a staff into a unit meeting to discuss patient satisfaction. During the meeting, several staff members assume control. The nurse manager does not intervene to regain control of the group. Which type of leadership style is the nurse embodying?

Laissez-faire

This leadership is most effective when all staff are clinical experts with a deep understanding of both clinical and administrative processes?

Laissez-faire

Surgical asepsis is defined as:

absence of all microorganisms.

A public health nurse is leaving the home of young mother who has a special needs baby. The neighbor states, "How is she doing, since the baby's father is no help?" What is nurse's BEST response to the neighbor?"

New mother's need support

The nurse is able to help promote safety and prevent injury by identifying which factors that have a direct impact on client safety? Select all that apply. Communication ability Community population Developmental level Mobility Type of health care facility

Mobility, Communication ability, and Developmental level. Rationale: Factors that impact safety include: 1. developmental level 2. lifestyle 3. mobility 4. sensory perception 5. knowledge level 6. communication ability 7. physical/psychosocial state.

When do we perform Hand hygiene?

Moment 1 - Before touching a patient Moment 2 - Before a clean or aseptic procedure Moment 3 - After a body fluid exposure risk Moment 4 - After touching a patient Moment 5 - After touching patient surroundings

Which is a task that the nurse can delegate to the unlicensed assistive personnel (UAP)?

Obtaining intake and output on a client with a foley catheter

An older adult has fluid volume deficit and needs to consume more fluids. Which approach by the nurse demonstrates gerontologic considerations?

Offer small amounts of preferred beverage frequently.

The nurse is assessing the wounds of patients. Which patients would be at most risk for delayed would healing.?

Older patient who is bed ridden patient with peripheral vascular disorder a patient who is obese a patient who is taking cortcosteriod drugs

The nurse cared for a client admitted with uncontrolled hypertension. The client suffered a stroke shortly after the nurse's shift ended. Which information will determine if the nurse is liable?

Omitting documentation of blood pressure at the end of the shift

A home care nurse discusses with a patient when visits will occur and how long they will last. In what phase of the helping relationship is this type of agreement established?

Orientation phase

What does PEARLA stand for?

P - presence E - empathy A - acknowledgement R - reflect L - listen A - ask questions

what is a powerful tool used for conflict engagement?

PEARLA

Which abbreviation is correct for use in documentation?

PO

A nurse notices a patient walking to the bathroom with a stooped gait, facial grimacing, and gasping sounds. Based on the these non-verbal clues, for which condition would the nurse assess?

Pain

An operating room nurse is caring for a client who will soon undergo an appendectomy. Which handwashing technique is most appropriate for the nurse to use when caring for this client?

Perform surgical hand scrub using detergent.

The nurse is caring for a client who has active tuberculosis and is under airborne precautions. The health care provider prescribes a computed tomography (CT) examination of the chest. Which action by the nurse is appropriate?

Place a surgical mask on the client and transport to the CT department at the specified time.

When a nurse observes that an older client's skin is dry and shiny and his nails are thickened, the nurse determines that the client is most likely experiencing Anemia Malnutrition Poor tissue perfusion Congestive heart failure

Poor tissue perfusion Explanation: Chronically poor perfusion may result in hair loss in the affected area, discolored skin, thickened nails, and shiny, dry skin indicative of inadequate tissue nutrition.

The nurse uses a small amount of sterile solution from a large, multiuse bottle to moisten gauze in a sterile field. What technique does the nurse use?

Pour the liquid into a sterile container within the sterile field.

Which principle should guide the nurse's documentation of entries on the client's health care record?

Precise measurements should be used rather than approximations.

A nurse is assessing the wound healing of a patient, documents that the wound formed a clean, straight line with little tissue loss. This would healed by:

Primary Intention

A nurse documents the following data in the client record according to the SOAP format: Client reports unrelieved pain; client is seen clutching the side and grimacing; client pain medication does not appear to be effective; Call in to primary care provider to increase dosage of pain medication or change prescription. This is an example of what charting method?

Problem-oriented method

The nurses at a health care facility were informed of the change to organize the clients' records into problem-oriented records. Which explanation could assist the nurses in determining the advantage of using problem-oriented records?

Problem-oriented recording emphasizes goal-directed care to promote the recording of pertinent data that will facilitate communication among health care providers.

A nurse assesses an area of pale white skin over a patient's coccyx. After turning the patient on her side, the skin becomes red and feels warm. What should the nurse do about these assessments?

Recognize that this is ischemia, followed by reactive hyperemia. Explanation: Blanching of skin over an area under pressure results from ischemia. When pressure is relieved, reactive hyperemia follows and the skin is red and feels warm. Reactive hyperemia is not a stage I pressure ulcer

In SBAR, what does R stand for?

Recommendations

Cardinal signs of infection are?

Redness Heat Swelling Spike in fever Pain loss of function

The nurse hears an unlicensed assitive personel (UAP) discussing a client's allergic reaction to a medication with another UAP in the cafeteria. What is the priority nursing action?

Remind the UAP about the client's right to privacy.

The nurse on a medical-surgical unit notices smoke from a client's room. Upon entering, the nurse notes that the curtain in the room is on fire. What should be the nurse's first action? Close the client's door. Activate the fire alarm. Remove the client from the room. Obtain the fire extinguisher.

Remove the client from the room. Rationale: In case of a fire, the nurse should (in this order) rescue anyone in immediate danger, activate the fire code system, notify the appropriate person, and confine the fire by closing doors and windows. Therefore, in this instance, the nurse's first action should be to remove the client from the room.

A client who was receiving care on a psychiatric unit committed suicide at a time when nurses are known to be handing off to nurses on the next shift. What is a responsibility of the organization when responding to this sentinel event? Inform local health care institutions about the event in order to promote safety. Change the institution's policies regarding supervision of clients. Appropriately discipline the nurses who were participating in the shift change. Report the event to the Joint Commission.

Report the event to the Joint Commission. Rationale: Sentinel events are serious safety events and need to be sent to the appropraite regulatory agency like the Joint Commission and to the state health agencies.

The nurse is sharing information about a client at change of shift. The nurse is performing what nursing action?

Reporting

The nurse is caring for a client who requests to see a copy of the client's own health care records. What action by the nurse is most appropriate?

Review the hospital's process for allowing clients to view their health care records.

A nurse is attempting to communicate with a patient who speaks a different language and doesn't not understand what is being communicated. What nursing action would best facilitate the communication process?

Speaking slowly and distinctly but not loudly.

4. While discussing home safety with the nurse, a patient admits that she always smokes a cigarette in bed before falling asleep at night. Which nursing diagnosis would be the priority for this patient? Impaired Gas Exchange related to cigarette smoking Anxiety related to inability to stop smoking Risk for Suffocation related to unfamiliarity with fire prevention guidelines Deficient Knowledge related to lack of follow-through of recommendation to stop smoking

Risk for Suffocation related to unfamiliarity with fire prevention guidelines Rationale: Smoking in bed is extremely dangerous, she could suffocate from fire while she sleeps.

The nurse is caring for a client who has an elevated temperature. When calling the health care provider, the nurse should use which communication tools to ensure that communication is clear and concise?

SBAR (ISBAR)

When recording data regarding the client's health record, the nurse mentions the analysis of the subjective and objective data, in addition to detailing the plan for care of the client. Which of the following styles of documentation is the nurse implementing?

SOAP charting

A nurse manager best demonstrates effective leadership characteristic by which action?

Sharing a vision for the unit and enlisting support

When attending a staff meeting a nurse is participating in what type of communication?

Small-group communication

The charge nurse is reviewing SOAP format documentation with a newly hired nurse. What information should the charge nurse discuss?

Subjective data should be included when documenting.

Which action by the nurse is compliant with the Health Insurance Portability and Accountability Act (HIPAA)?

Submitting a written notice to all clients identifying the uses and disclosures of their health information

The nurse planning to insert an indwelling urinary catheter into a client should utilize which technique?

Surgical asepsis

The nurse is working at a local elementary school. A mother arrives to pick up her 6-year-old son and has her 2-year-old daughter in tow. Based on the nurse's developmental knowledge of toddlers, which behavior would most concern the nurse? The 2-year-old helping mom to open the front door of the school. The 6-year-old riding a bike on the playground with his friend. The 2-year-old leaning against the screen of a window in a classroom. The 2-year-old and 6-year-old each holding the mother's hand.

The 2-year-old leaning against the screen of a window in a classroom. Rationale: Windows are a concern for toddlers.

A client has been diagnosed with PVD. On which area of the body should the nurse focus the assessment?

The lower extremities

When a nurse is planning for learning, who must decide who should be included in the learning sessions?

The nurse and the client

A nurse is filing a safety event report for an older adult client who tripped and fell when getting out of bed. Which action exemplifies an accurate step of this process? The nurse adds the information in the safety event report to the client health record. The nurse calls the primary health care provider to fill out and sign the safety event report. The nurse provides an opinion of the physical and mental condition of the client that may have precipitated the incident. The nurse details the client's response and the examination and treatment of the client after the incident.

The nurse details the client's response and the examination and treatment of the client after the incident. Rationale: Unintentional injury/incident that compromises safety in a health care agency needs a safety event report (incident report). -These safety event/incident reports do NOT need to be part of the medical record of patients and should not be reported in documentation. -HCP does not fill out these forms unless they were witnesses -No opinions just factual information

The nurses on a busy surgical ward use hand hygiene when caring for postsurgical patients. Which action represents an appropriate use of hand hygiene?

The nurse keeps fingernails less than 1/4 in (0.63 cm) long.

The nurse is caring for a penrose drain. What nursing action reflects a step to care for the penrose drain that needs to be shortened everyday.

The nurse pulls the drain out a short distance using sterile scissors and a twisting motion and cuts off the end of the drain with sterile scissors.

A nurse is completing an intake assessment. The nurse notes that an older adult male client appears to have bruises in varying stages of healing. Which action by the nurse indicates an understanding of her responsibilities? The nurse should notify the primary care physician about the bruises. The nurse should contact the facility's social services department. The nurse should question the client about the source of the bruises. The nurse should request permission from the client to photograph the bruises.

The nurse should question the client about the source of the bruises. Rationale: The first thing you should do is assess and question the source of the bruises. If the nurse feels like there is a potential abuse situation, then the nurse needs to report it.

The nurse is caring for a patient who has a pressure ulcer on his back. What nursing intervention would the nurse perform?

The nurse uses positioning devices and techniques to maintain posture and distribute weight evenly for the patient in a chair

The nurse caring for a postoperative patient is cleaning the patients wound. Which nursing action reflects the proper procedure?

The nurse works outward from the wound in lines parallel to it.

Which technique would a nurse employ when using listening skills appropriately?

The nurse would listen to the themes in the patient's comments.

Which factor is related to the highest proportion of falls in long-term care settings? Toileting Agitation Polypharmacy Impaired sleep patterns

Toileting Rationale: 42% of falls were related to toileting, often involving getting out of bed or walking to the bathroom.

A female patient who is being treated for self inflicted wounds tells the nurse that she is anorexic. What criteria would alert the health care worker to her nutritional risk?

Total lymphocyte count of 1,500/mm3

A nurse is working as part of a group on developing strategies to meet the health care needs of the children being served by the facility. The nurse identifies the need for expanded play therapy programs. The nurse works to motivate others in the group and promote a common vision. The nurse and the group communicate openly and honestly with the others involved, focusing on the process as well as the outcomes. The nurse is exhibiting which style of leadership?

Transformational

A client has requested a translator to help understand the questions that the nurse is asking during the client interview. The nurse knows that what is important when working with a client translator?

Translators may need additional explanations of medical terms.

The nurse s giving a back rub to a patient and notices a stage 2 pressure ulcer. What should the nurse do next?

Use normal saline to clean the pressure ulcer

While walking in the woods, an 8-year-old boy trips and a stick cuts his right leg. The camp nurse inspects the wound and determines a portion of the dermis is intact, so she cleanses and bandages the wound. What wound classification will the nurse document on the child's health record?

Unintentional, partial-thickness wound. Explanation: An unintentional wound is an accidental wound. A partial-thickness wound is characterized by all or a portion of the dermis remaining intact

A new graduate is working at a first job. Which statement is most important for the new nurse to follow?

Use abbreviations approved by the facility.

A nurse is caring for a 78 year old client who was admitted after a femur fracture. The primary care provider placed the client on bed rest. Which of the following actions should the nurse perform to prevent a pressure ulcer?

Use pillows to maintain a side lying postion as needed

During data collection, the nurse auscultates low-pitched, soft sounds over the lungs' peripheral fields. Which appropriate terminology would the nurse use to describe these lung sounds when documenting? Crackles Bronchovesicular Bronchial Vesicular

Vesicular Explanation: Vesicular breath sounds are normal and described as low-pitched, soft sounds over the lungs' peripheral fields. Crackles are soft, high-pitched, discontinuous popping sounds heard on inspiration. Medium-pitched blowing sounds heard over the major bronchi describe bronchovesicular breath sounds. Bronchial breath sounds are loud, high-pitched sounds heard over the trachea and larynx.

A nurse assessing a client's respiratory status gets a weak signal from the pulse oximeter. The client's other vital signs are within reference ranges. What is the nurse's best action? Warm the client's hands and try again. Place the probe on the client's earlobe. Shine available light on the equipment to facilitate accurate reading. Use a blood pressure cuff to increase circulation to the site.

Warm the client's hands and try again. Explanation: Finding an absent or weak signal, the nurse should check vital signs and client condition. If satisfactory, warming the extremity may facilitate a stronger reading. This should be attempted prior to resorting to using the client's earlobe. Bright light can interfere with the operation of light sensors and cause an unreliable report. A blood pressure cuff will compromise venous blood flow to the site leading to inaccurate readings.

The nurse is teaching a nursing student about proper latex glove use. Which teaching will the nurse include? Use hand cream or lotion after removing gloves to preserve skin integrity. Wash hands thoroughly after removing gloves with a pH balanced soap. Snap the gloves when applying them to ensure proper fit. Use powdered gloves.

Wash hands thoroughly after removing gloves with a pH balanced soap. Rationale: Use pH balanced soap after using latex gloves.

A patient states, "I have been experiencing complications of diabetes." The nurse needs to direct the patient to more information. What is the MOST appropriate comment or question to elicit additional information?

What specific complications have you experienced?

During rounds, a charge nurse hears the patient care technician yelling loudly to a patient regarding a transfer from the bed to chair. Upon entering the room, what is the nurse's BEST response?

When your patient is safe and comfortable, meet me at the desk.

Which is a common anion?

Which is a common anion? Cl- Mg+, K+, and Ca+

You are applying a saline-moistened dressing to a client's wound. The client asks, "Wouldn't it be better to let my wound dry out so a scab can form?" Which of the following responses is most appropriate?

Wounds heal better when a moist wound bed is maintained." Explanation: A moist wound surface enhances the cellular migration necessary for tissue repair and healing.

A nurse is documenting care in a source-oriented record. What action by the nurse is mostappropriate?

Write a narrative note in the designated nursing section.

The acute care nurse is caring for a client whose large surgical wound is healing by secondary intention. The client asks, "Why is my wound still open? Will it ever heal?" Which of the following responses by the nurse is most appropriate?

Your wound will heal slowly as granulation tissue forms and fills the wound.

A nurse is using time management techniques when planning activities for patients. Which nursing action reflects effective time management? a. The nurse asks patients to prioritize what they want to accomplish each day b. The nurse includes a "nice to do" for every "need to do" task on the list c. The nurse "front-loads" the schedule with "must-do" priorities d. The nurse avoids helping other nurses if scheduling does not permit it

a

The nurse is irrigating a nasogastric tube attached to suction and finds that the flush solution is meeting a lot of force when plunger is pushed. what would be the nurses first intervention in this situation? a) inject 20 to 30 mL of free air into the abdomen in attempt to reposition the tube and enable flushing of the tube b) check the suction canister to ensure that the suction is working appropriately c) assess the abdomen for distention and ask the patient if he or she is experiencing any nausea or any abdominal discomfort

a

A nurse is arranging for home care for clients and reviews the Medicare reimbursement requirements. Which client meets one of these requirements?

a client who is homebound and needs skilled nursing care

During his stay in the hospital, a male client has established a pattern of maintaining urinary continence during the day, but he is experiencing incontinence at night. What intervention should the nurse implement in this client's care? A) condom catheter B) indwelling catheter C) intermittent catheterization at bedtime D) toileting the client every 2 hours

a) condom catheter

Which statements about suprapubic catheters is true? a)They are often preferred over an indwelling urethral catheter for long-term urinary drainage. b)They are surgically inserted through a small incision above the umbilicus. c) They drain urine directly from the ureters. d) Inadvertent dislodgement can permanently damage the urethra.

a)They are often preferred over an indwelling urethral catheter for long-term urinary drainage.

A nurse is a servant leader working in an economically depressed community to set up a free mobile health clinic for the residents. Which actions by the leader BEST exemplify a key practice of servant leaders? Select all that apply a. The nurse motivates coworkers to solicit funding to set up the clinic. b. The nurse sets only realistic goals that are present oriented and easily achieved. c. The nurse forms an autocratic governing body to keep the project on track. d. The nurse spends time with supporters to help them grow in their roles. e. The nurse first ensures that other's lowest priority needs are served. f. The nurse prizes leadership because of the need to serve others.

a, d, f

conflict engagement

address conflict in the workplace (rather than avoiding conflict)

A client with dehydration will have an increase in:

aldosterone

laissez-faire leadership

allows the group to function more or less on its own

After a nurse manager implements a solution to the problem of delays in obtaining supplies, the first task for the manager is to:

assess whether the desired results have occurred.

After performing neuro checks every 2 hours after an ischemic stroke, the nurse determines the client's neurological status is deterioriating. Which nursing action best prevents an adverse outcome in this client's care? initiating rehabilitation as soon as possible arranging for STAT computed tomography of the client's head documenting any changes in the client's status in a detailed and timely manner assessing the client's neurological status more frequently than ordered

assessing the client's neurological status more frequently than ordered Rationale: Assessing a patient more often can prevent adverse outcomes.

magnet status facilities do what?

attract and retain well-qualified nurses who promote quality patient care

A new nurse manager at a small hospital is interested in achieving Magnet status. Which action would help the hospital to achieve this goal? a. Centralizing the decision-making process b. Promoting self-governance at the unit level c. Deterring professional autonomy to promote teamwork d. Promoting evidence-based practice over innovative nursing practice

b

A nurse manager who is attempting to institute the SBAR process to communicate with health care providers and transfer patient information to other nurses is meeting staff resistance to the change. Which action would be most effective in approaching this resistance? a.Containing the anxiety in a small group and moving forward with the initiative b.Explaining the change and listing the advantages to the person and the organization c.Reprimanding those who oppose the new initiative and praising those who willingly accept the change d.Introducing the change quickly and involving the staff in the implementation of the change

b

The nurse is inserting a rectal tube to administer a large-volume enema. which nursing action is performed correctly in this procedure? a) position the patient on his or her back and drape properly b) slowly and gently insert the enema tube 3 to 4 in for an adult c) introduce solution quickly over a period of 3 to 5 minutes

b

The nurse is scheduling tests for patient who is experiencing bowel alterations. What is the most logical sequence of tests to ensure an accurate diagnosis? a) barium studies, endoscopic examination, decal occult blood test b) fecal occult blood test, barium studies, endoscopic examination c) barium studies, decal occult blood test, endoscopic examination

b

A nurse uses a portable bladder ultrasound device to assess bladder volume for a client who is unable to void. Which statement accurately details information needed to interpret the results? a) The scan is contraindicated for a female who had a hysterectomy. b) The device must be programmed for the gender of the client by pushing the correct button on the device. c) A postvoid residual (PVR) volume more than 50 mL indicates adequate bladder emptying. d) A postvoid residual (PVR) volume of less than 150 mL is often recommended as the guideline for catheterization, because this has been associated with the development of urinary tract infections.

b) The device must be programmed for the gender of the client by pushing the correct button on the device.

During data collection, the client expresses concern over a change in the color of the urine from tea-colored to green since beginning a new medication. Which appropriate question would the nurse ask this client? a)"Are you taking a diuretic?" b)"Are you taking any B-complex vitamins?" c)"Are you taking phenazopyridine (Pyridium)?" d) "Are you taking levodopa (L-dopa)"

b)"Are you taking any B-complex vitamins?"

A nurse manager is attempting to update a health care provider's office from paper to electronic health records (EHR) by using the eight-step process for planned change. Place the following actions in the order in which they should be initiated: a. The nurse devises a plan to switch to EHR. b. The nurse records the time spent on written records versus EHR. c. The nurse attains approval from management for new computers. d. The nurse analyzes all options for converting to EHR. e. The nurse installs new computers and provides an in-service for the staff. f. The nurse explores possible barriers to changing to EHR. g. The nurse follows up with the staff to check compliance with the new system. h. The nurse evaluates the effects of changing to EHR.

b, f, d, c, a, e, h, g

The nurse is administering psyllium to a patient with constipation. What mechanism of action would the nurse expect from this drug? a) chemical stimulation of peristalsis b) softening of the fecal material c) increasing intestinal bulk to enhance mechanical stimulation of the intestine d) drawing water into the intestines to stimulate peristalsis

c

The nurse is caring for a patient who is scheduled for an esophagogastroduodenoscopy (EGD). What action would the nurse take to prepare the patient for this procedure? a) ensure that the patient ingests a gallon of bowel cleanser, such a GoLytely, in a short period of time b) inform patient that a chalky-tasting barium contrast mixture will be given to drink before the test c) ensure that the patient fasts 6 to 12 hours before the test as per policy

c

An RN on a surgical unit is behind schedule administering medications. Which of the RN's other tasks can be safely delegated to a UAP? a. The assessment of a patient who has just arrived on unit b. Teaching a patient with newly diagnosed diabetes about foot care c. Documentation of a patient's I & O on the flow chart d. Helping a patient who has recently undergone surgery out of bed for the first time

c What they can delegate are assistance with basic care activities (bathing, grooming, ambulation, feeding) and things like taking vital signs, measuring intake and output, weighing, simple dressing changes, transfers, and post mortem care.

A nurse is asked to act as a mentor to a new nurse. Which nursing action is related to this process? a. The nurse mentor accepts payment to introduce the new nurse to his or her responsibilities b. The nurse mentor hires the new nurse and assigns duties related to the position d. The nurse mentor makes it possible for the new nurse to participate in professional organizations e. The nurse mentor advises and assists the new nurse to adjust to the work environment of a busy emergency department

d

The nurse is administering a large-volume cleansing enema to a patient. Which nursing action is performed correctly? a) the nurse places the patient on bedpan in the supine position while receiving enema. b) the nurse uses cool tap water for the enema solution c) the nurse elevates solution to no higher than 10 in above level of anus d) the nurse gives the solution slowly over a period of 5 to 10 minutes

d

which situations require autocratic leadership?

emergency situations

democratic leadership

equality between the leader and followers

The residential home nurse is caring for a client who lives in an assisted living unit. In designing a plan of care to prevent fires, the nurse identifies which as the highest risk to the client? clothes dryer gas stove electrical sockets cigarette smoking

gas stove Rationale: (ugh this was my first choice), cooking causes the MOST residential fires. MOST fire deaths occur in the home just fyi

The nurse is conducting a home care visit for a new mother who delivered a baby 3 days ago. Which finding within the home requires immediate nursing intervention? hot water heater thermostat set at 130 degrees F (54.4 degrees C) electrical outlets have covers over them one fire extinguisher is noted in the kitchen infant's sleepwear is made from flame-resistant fabrics

hot water heater thermostat set at 130 degrees F (54.4 degrees C) Rationale: Hot water heater thermostat is set abover 120 degrees which can burn the infant's skin.

Examples of Surgical Asepsis (sterile technique)

inserting an undwelling urinary cath inserting an IV cath

When charting the assessment of a client, the nurse writes, "Client is depressed." This documentation is an example of:

interpretation of data.

Medical asepsis (clean technique)

involves procedures used to reduce and prevent the spread of microorganisms.

When a manager identifies problems, developing goals, objectives, and related strategies to meet the demands of the clinical site they are in what role?

planning

Secondary Intention

large wounds with considerable tissue loss

implied power

power obtained by force of a person's personality

explicit power

power obtained by virtue of a person's position

Surgical technique (sterile technique)

practices used to render and keep objects and areas free of microorganisms

organizations use transactional leadership to

recognize employees' progress in meeting pre-established goals and work deadlines.

A decrease in arterial blood pressure will result in the release of:

renin.

The nurse is caring for a client who has been repetitively pulled at IV lines and the urinary catheter. After other methods of diverting the client's behaviors fail to prevent this behavior, and chemical restraints fail, which treatment does the nurse anticipate will be ordered? temporary application of devices that reduce the client's ability to move arms providing a sleep agent to help the client rest instead of pulling IV lines and the catheter delegating to the unlicensed assistive personnel (UAP) to sit with the client administration of an antipsychotic agent to alter the client's behavior

temporary application of devices that reduce the client's ability to move arms Rationale: After diversion behaviors and chemical (drug) restraints have failed, the nurse should use temp devices to reduce the client's ability to move arms

In a decentralized management structure who makes the decisions?

those who are most knowledgeable

A nurse is caring for a 55-year-old postoperative client. The client returns to the ICU after surgery intubated and mechanically ventilated with a Salem sump nasogastric tube, a Foley catheter, and a PICC line in place. Based on the nurse's knowledge of the most common hospital-acquired infections, which apparatus is most important to remove first?

urinary catheter

The nurse is preparing a SOAP note. Which assessment findings are consistent with objective client data?

urine output 100 ml

Which is not appropriate regarding the use of gowns as PPE?

use of one gown per person per shift

Which safety tip could the nurse give to parents to help decrease the risk of the leading cause of injury or death in children 1 to 4 years of age? "Store medications in a locked area to prevent children from getting into them." "Never smoke in the bed in the house when young children are present." "Always provide close supervision for young children when they are in or around pools and bathtubs." "Never keep firearms in the home with young children."

"Always provide close supervision for young children when they are in or around pools and bathtubs." Rationale: Leading cause of death of children 1-4 is drowning

The nurse is teaching the client with a pulmonary disorder about deep breathing. The client asks, "Why is it important to start by breathing through my nose, then exhaling through my mouth?" Which appropriate response would the nurse give this client? "Breathing through your nose first encourages you to sit up straighter to increase expansion of the lungs during inhalation." "Breathing through your nose first will warm, filter, and humidify the air you are breathing." "If you breathe through the mouth first, you will swallow germs into your stomach." "We are concerned about you developing a snoring habit, so we encourage nasal breathing first."

"Breathing through your nose first will warm, filter, and humidify the air you are breathing." Explanation: Nasal breathing allows the air to be warmed, filtered, and humidified. Nose breathing does not encourage the client to sit up straight. The purpose of nasal breathing is not to prevent germs from entering the stomach or to discourage snoring.

The nurse is caring for a client with Alzheimer's disease. A family member states, "I am afraid I will go to bed one night, and the next morning my loved one will be missing from wandering off." What is the appropriate nursing response? "Consider the Alzheimer's Association 'Safe Return' program." "Adjust sleeping schedules so that you can monitor your loved one as they sleep." "Clients with Alzheimer's disease often wander." "I know, my parent has Alzheimer's disease and I worry about that too."

"Consider the Alzheimer's Association 'Safe Return' program." Rationale: The appropriate nursing response is to refer the client's family member to a program such as the Alzheimer's Association's "Safe Return" program. This validates the family member's concern, and provides a resource

The nurse is caring for a client with Alzheimer's disease. A family member states, "I am afraid I will go to bed one night, and the next morning my loved one will be missing from wandering off." What is the appropriate nursing response? "Clients with Alzheimer's disease often wander." "Consider the Alzheimer's Association 'Safe Return' program." "Adjust sleeping schedules so that you can monitor your loved one as they sleep." "I know, my parent has Alzheimer's disease and I worry about that too."

"Consider the Alzheimer's Association 'Safe Return' program." Rationale: Giving family member resource.

A 36-year old patient who underwent a hysterectomy 4 days ago says to the nurse, "I wonder if I'll still feel like a women." Which response would most likely encourage the patient to expand on this and express her concerns in more specific terms?

"Feel like a women..."

The student nurse asks, "what is interstitial fluid?" What is the appropriate nursing response?

"Fluid in the tissue space between and around cells." Intracellular fluid (fluid inside cells) represents the greatest proportion of water in the body. The remaining body fluid is extracellular fluid (fluid outside cells). Extracellular fluid is further subdivided into interstitial fluid (fluid in the tissue space between and around cells) and intravascular fluid (the watery plasma, or serum, portion of blood).

The nurse is preparing to administer granulocytes to a client admitted with a severe infection. Which teaching by the nurse is most appropriate?

"Granulocytes are a type of white blood cell that can help fight infection."

A large health care organization has committed to promoting a just culture when adverse events and near misses take place. Which question will guide the organization's response when a nurse commits an error? "How did the nurse's actions contribute to this error?" "How have other organizations responded to nurses in events like this?" "Have the client and the family been informed about this?" "What is the organization's legal liability in this matter?"

"How did the nurse's actions contribute to this error?" Rationale: Key to just culture is a recognition that not all errors are the same, and that nurses' contribution to errors vary greatly.

Which statement made by a client who was recently admitted to the medical unit with a diagnosis of pneumonia indicates a physical inability to learn?

"I am having difficulty breathing."

A staff nurse is talking with a clinical nurse leader and asks, "What exactly do you do?" Which statement by the clinical nurse leader wouid be appropriate?

"I collaborate with health care teams to promote client care."

The nurse is preparing discharge teaching for a client who has chronic obstructive pulmonary disease (COPD). Which teaching about deep breathing will the nurse include? "Take in a little air over 10 seconds, hold your breath 15 seconds, and exhale slowly." "Take in a small amount of air very quickly and then exhale as quickly as possible." "Inhale slowly over three seconds, purse your lips, contract abdominal muscles, and exhale slowly." "Take in a large volume of air over 5 seconds and hold your breath as long as you can before exhaling."

"Inhale slowly over three seconds, purse your lips, contract abdominal muscles, and exhale slowly." Explanation: Pursed-lip breathing is a form of controlled ventilation that is effective for clients with COPD. Other answers are incorrect techniques for deep breathing.

A nurse asks a nurse manager why staff nurses on the unit cannot document in a separate record (instead of the client record) to make it easier to find information on nursing-specific actions. What is the best response by the nurse?

"Legal policy requires nursing practice to be permanently integrated into the client record."

The nurse is caring for a client who will be undergoing surgery in several weeks. The client states, "I would like to give my own blood to be used in case I need it during surgery." What is the appropriate nursing response?

"Let me refer you to the blood bank so they can provide you with information."

Which statement indicates that a family understands the teaching that has been provided by the nurse related to car seat safety for a 9-month-old infant? "We place our baby in a front-facing car seat in the middle of the back seat of the car." "We place our baby in a front-facing car seat in the front of the car so that he doesn't cry." "We place our baby in a rear-facing car seat in the back seat of the car." "We place our baby in a rear-facing car seat in the front of the car so that we can see him in case he chokes.

"We place our baby in a rear-facing car seat in the back seat of the car." Rationale: Infants until the age of 2 should be in the rear-facing infant seat in the back seat of the care until they reach the max height and weight for a front-facing child car seat.

The nurse calls the health care provider due to changes in the client's status. Using the SBAR, the nurse is about to address Recommendation. Which statement appropriately supports this part of the SBAR?

"Will you prescribe a complete blood count to check the white blood cell count and a culture?"

Which clients are most at risk for falling due to altered mobility? Select all that apply. A client with a spinal cord injury A middle-aged woman who had surgery 2 weeks ago and wears high heels All older adults An older adult client with an unsteady gait A client who requires crutches in unfamiliar health care settings

1. A client with a spinal cord injury 2. An older adult client with an unsteady gait 3. A client who requires crutches in unfamiliar health care settings Rationale: Not the middle-aged woman because Prep U says they aren't most at risk for falling (bih what lol). -Limitations in mobility are unsafe and cause client injury. -Remember not to choose answers with "all", "always", "never" in them lol

A nurse is performing safety assessments in a health care facility. Which statements reflect considerations a nurse should keep in mind when assessing a client for safety? Select all that apply. A person with a history of falls is likely to fall again. Some people are more at risk for accidents than others. Fires are responsible for most hospital incidents. Between 15% and 25% of falls result in fractures or soft tissue injury. A medication regimen that includes diuretics or analgesics places an individual at risk for falls. A nurse whose behavior is reasonable and prudent, and similar to what would be expected of another nurse in a similar circumstance, is still likely to be found liable if a client falls, especially if an injury results.

1. A person with a history of falls is likely to fall again. 2. Some people are more at risk for accidents than others. 3. A medication regimen that includes diuretics or analgesics places an individual at risk for falls. Rationale: -Person with hx of fall are at risk for falls in the future. -Some people are more careless and more at risk for falls -FALLS not fires are responsible for most hospital incidents. -~33% of falls result in fx or soft issue injury -A med regimen with diuretics or analgesics (causes unsteady gait due to drowsiness) = risk for falls

The nurse is caring for a client with a latex allergy. Which nursing interventions are appropriate? (Select all that apply.) Communicate to the interdisciplinary healthcare team to use nonlatex equipment. Teach client to wear Medic-Alert bracelet. Remove blueberries from the client's dietary tray. Assign client to a semi-private room so roommate can report any reactions. Flag the chart and room door. Apply an allergy-alert identification bracelet on the client.

1. Communicate to the interdisciplinary healthcare team to use nonlatex equipment. 2. Teach client to wear Medic-Alert bracelet. 3. Flag the chart and room door. 4. Apply an allergy-alert identification bracelet on the client. Rationale: Chart and room should reflect patient's latex allergy. Other healthcare personnel should know about the allergy as well. The nurse will teach the client to wear a Medic-Alert bracelet at all times for safety purposes. Used to identify important medical information

The nurse must apply a hospital gown to a patient receiving an intravenous infusion in the forearm. The nurse should: 1. Insert the IV bag and tubing through the sleeve from inside of the gown first 2. Disconnect the IV at the insertion site, apply the gown, and then reconnect the IV 3. Close the clamp on the IV tubing no more than 15 seconds while putting on the gown 4. Don the gown on the arm without the IV, drape the gown over the other shoulder, and adjust the closure behind the neck

1. Insert the IV bag and tubing through the sleeve from inside of the gown first Rationale: Prevents tension on the IV and limits unnecessary exposure to enfection

The nurse is planning care for a patient with a wrist restraint. The restraint should be removed, the area massaged, and the joints moved through their full range every: 1. Shift 2. Hour 3. Two hours 4. Four hours

3. Two hours

The nurse is caring for a confused patient. To prevent this patient from falling, the nurse should: 1. Encourage the patient to use the corridor handrails 2. Place the patient in a room near the nurses' station 3. Reinforce how to use the call bell 4. Maintain close supervision

4. Maintain close supervision Rationale: Confused patients may not be able to understand you when you encourage, reinforce them, or even when you place them closer to the nurses' station. So you can't leave them by themselves.

Unintentional injuries are a major cause of disability and death in the United States. For adults, where do unintentional injuries fall on the list of leading causes of death? Fifth Tenth First Eighth

5th Rationale: After heart disease, cancer, stroke, and chronic obstructive lung disease.

A physician has asked the nurse to use microdrip tubing to administer a prescribed dosage of the IV solution to a client. WHat is the standard drop factor of microdrip tubing?

60 drops/mL

A nurse is counseling several clients for depression. Four of them do not seem to be improving, which leads the nurse to suggest a referral to a psychiatric nurse practitioner. Which of these clients would be most likely to attend the scheduled appointment?

A 28-year-old female who works nights, is willing to try, and asks about insurance coverage of the appointment

When performing fall risk assessments, which client does the nurse determine is most at risk for falls? A 50-year-old male being cared for in an unfamiliar health care environment A 60-year-old male with weakness in his left side and slowed reaction time A 70-year-old female with postural hypotension who wears eyeglasses, but has no history of falls An 80-year-old female with a history of falling last year and breaking a hip

A 70-year-old female with postural hypotension who wears eyeglasses, but has no history of falls Rationale: This patient has 3 RISK FACTORS These are the risk factors: 1. Being over 65 2. Hx of falls 3. Impaired vision or sense of balance 4. Postural hypotension 5. Altered posture or gait 6. Meds = diuretics (pee more often), tranquilizers, sedatives, hypnotics, or analgesics (makes you sleepy) 7. Slowed reaction time 8. Confusion/disorientation 9. Impaired mobility 10. Weakness/physical frailty 11. Unfamiliar environment

In which client should the nurse prioritize assessments for respiratory depression? A client taking a beta-adrenergic blocker for hypertension A client taking antibiotics for a urinary tract infection A client taking insulin for type 1 diabetes A client taking opioids for cancer pain

A client taking opioids for cancer pain Explanation: Many medications affect the function of the respiratory system and depress the respiratory system. The nurse should monitor clients taking certain medications, such as opioids, for rate and depth of respirations. Beta-adrenergic blockers, antibiotics, and insulin do not appreciably affect the respiratory system.

The nurse caring for patients in a long-term care facility knows that there are factors that place certain patients at a higher risk for falls. Which patients would the nurse consider to be in this category? Select all that apply. A patient who is older than 60 years A patient who has already fallen twice A patient who is taking antibiotics A patient who experiences postural hypotension A patient who is experiencing nausea from chemotherapy A 70-year old patient who is transferred to long-term care

A patient who has already fallen twice, A patient who experiences postural hypotension, A 70-year old patient who is transferred to long-term care Rationale: Risk factors for falls include a) patients over 65+, b) documented hx of falls, c) postural hypotension, and d) unfamiliar environment. Meds including: diuretics, tranquilizers, sedatives, hypnotics, or analgesics are also risk factors, but NOT chemo or antibiotics.

A client in a long-term care facility becomes confused and disoriented at night and is incontinent during these periods of confusion due to the inability to find the commode. During the day, the client does not experience confusion and is continent. What type of incontinence is this client experiencing during the nighttime hours? A) functional incontinence B) transient incontinence C) stress incontinence D) reflex incontinence

A) functional incontience

The nurse collects a urine sample from a client for urinalysis. What would the nurse document as a normal characteristic? A) light yellow color B) cloudy appearance C) presence of mucus shreds D) ammonia odor

A) light yellow color

The nurse is preparing to catheterize a client who is incontinent of urine following bladder surgery. What fact should the nurse keep in mind when performing catheterization? keep in mind when performing catheterization? a) The bladder normally is a sterile cavity. b) The external opening to the urethra should always be sterilized. c) Pathogens introduced into the bladder remain in the bladder. d) A normal bladder is as susceptible to infection as an injured one.

A) the bladder normally is a sterile cavity

Which nursing diagnosis would the nurse make based on the effects of fluid and electrolyte imbalance on human functioning?

Acute Confusion related to cerebral edema

When preparing client teaching materials, how does the nurse best assess a client's preferred learning style?

Ask the client, "Do you learn best by observing, valuing, or doing?"

A nurse states the following to another nurse who is constantly forgetting to wash her hands between patients: "It looks like you keep forgetting to wash your hands between patients. Its really not safe your patients. Let's think of some type of reminder we can use to help you remember." This communications is an example of what type of speech?

Assertive

A client is received into the emergency department after getting shot in the chest. The client is hemorrhaging profusely and is in hypovolemic shock. The nurse calls a code blue. What type of leadership style will be most effective during the management of the code?

Autocratic leadership

What is the most important safety concept that a nurse should include in the teaching plan for a family with a newborn infant in the household? Avoid stuffed animals and blankets in the crib. Educate about, and be aware of, signs of risky behaviors. Include safeguards to prevent falls in the home. Teach seat belt safety.

Avoid stuffed animals and blankets in the crib. Rationale: Newborns esp before the age of 4 months are prone to suffocation. Toddlers and older children = falls. Teens = Risky behaviors.

when an alcohol-based handrub can be used to decontaminate hands (IHI, 2018):

Before direct contact with patients After direct contact with patient's skin After contact with body fluids, mucous membranes, nonintact skin, and wound dressings, if hands are not visibly soiled After removing gloves Before inserting urinary catheters, peripheral vascular catheters, or invasive devices that do not require surgical placement Before donning sterile gloves prior to an invasive procedure (e.g., inserting a central intravascular catheter) If moving from a contaminated body site to a clean body site during patient care After contact with objects (including equipment) located in the patient's environment

Upon review of a postoperative patient's medication list, the nurse recognizes that which medication will delay the healing of the operative wound?

Corticosteroids Patients who are taking corticosteroids or require postoperative radiation therapy are at high risk for delayed healing and wound complications. Corticosteroids decrease the inflammatory process, which may delay healing

A nurse is caring for a client on a medical surgical unit who has had an evisceration of an abdominal wound after a coughing episode. Which of the following actions by the nurse are appropriate in this situation? Select all that apply.

Cover wound with gauze place supine postion Use sterile techniques

The nurse documents a progress note in the wrong client's electronic medical record (EMR). Which action would the nurse take once realizing the error?

Create an addendum with a correction.

A client is suffering from infectioius diarrhea, dehydration and right sided paralysis is confined to bed. What is the client most prone to?

Decubitus Ulcer

The parents of a school-age child are meeting with the nurse for health promotional education for their child. The child has the following assessment data: a 7-year-old male with diabetes mellitus type 1 with a hemoglobin A1C level of 8.3%, a body mass index (BMI) of 31.7, and a BMI percentile of 99. What are the most appropriate learning diagnoses for this first session?

Deficient Knowledge: Imbalanced nutrition: more than body requirements, and ineffective health maintenance.

A postoperative client is being transferred from the bed to a gurney and states, "I feel like something has just given away." What should the nurse assess in the client?

Dehiscence of the wound Dehiscence is the partial or total separation of wound layers as a result of excessive stress on wounds that are not healed. Clients at greater risk for these complications include those who are obese or malnourished, smoke tobacco, use anticoagulants, have infected wounds, or experience excessive coughing, vomiting, or straining. An increase in the flow of fluid from the wound between postoperative days 4 and 5 may be a sign of an impending dehiscence. The client may say that "something has suddenly given way." If dehiscence occurs, cover the wound area with sterile towels moistened with sterile 0.9% sodium chloride solution and notify the physician. Once dehiscence occurs, the wound is managed like any open wound. Manifestations of infection include redness, warmth, swelling, and heat. With herniation, there is protrusion through a bodily opening. Evisceration is a term that describes protrusion of intra-abdominal contents

A couple has just learned that their newborn infant has a congenital cardiac anomaly that will require many lifestyle modifications, surgical corrections, and hospital stays. Place the following aspects of the couple's client education in the correct order that nurses should conduct them.

Determine their emotional readiness to learn. Draft learning outcomes. Select educational strategies. Implement various educational techniques. Revise the learning plan if needed.

The nurse caring for a patient who is hospitalized following a double mastectomy is preparing a discharge plan for the patient. What should be the focus of the termination phase?

Determining the progress make in achieving established goals.

During an interaction with a patient diagnosed with epilepsy, a nurse notes that the patient is silent after communicating the nursing care plan. What would be appropriate nurse responses in this situation?

Discuss the silence with the patient to ascertain its meaning. Allow the patient time to think and explore inner thoughts. Determine if the patient's culture requires pauses between conversion.

Which skin disorder is associated with asthma? Seborrhea Psoriasis Abrasions Eczema

Eczema Explanation: The client with asthma often recalls childhood allergies and eczema.

The designated charge nurse on the telemetry unit organizes and facilitates the unit meetings, makes assignments, and performs staff evaluations. This nurse's power to lead is of which type?

Explicit

Which method of charting did the nurse use to document "Fluid Volume Overload. On assessment client's lower limbs edmatous ++. Affected leg elevated and furosemide 40 mg intramuscular given. No signs of deep vein thrombosis noted. Limbs now edema +"?

FOCUS

As the nurse enters the room to teach the client about self-care at home, the client states, "I am glad you are here. I need some pain medicine. I can't stand it anymore." What is the best action of the nurse?

Have the client rate pain level, and reschedule the teaching session.

A nurse is part of a group working at an acute care facility. The group wants to implement strategies toward achieving magnet status for the facility. When describing their plan to the organization, which information would they include as a benefit to the organization? Select all that apply.

Improved client safety Collaborative practice Financial growth of the organization Retention of highly talented individuals

An older adult client who is recovering from a stroke is scheduled to be transferred to the rehabilitation unit in the morning. The client is tearful and reports feeling lonely and abandoned in the hospital unit. The family visits daily, and flowers and cards are in the room. Documentation in the chart indicates that the client's pastor has been by twice in the past week to visit. Which nursing diagnosis and outcome criteria need to be addressed immediately for this client?

Ineffective Coping; verbalizes support systems.

The unlicensed assistive personnel (UAP) tells the nurse that a client is very confused and trying to get out of bed without assistance. What is the appropriate action by the nurse? Contact the physician for a restraint order. Initiate use of a bed alarm. Administer the client's sedative as ordered. Put up all four side rails on the bed.

Initiate use of a bed alarm. Rationale: Prevent clients from getting out of bed using the least restrictive method possible first. All four side rails and chemical restraints like sedatives are considered forms of restraints, they should be used as a last resort if the patient pay harm themselves or others.

This leadership is used when a leader wants the problem to be solved completely by team members

Laissez-faire

Which style of leadership is rarely used in a hospital setting because of the difficulty of task achievement by independent nurses?

Laissez-faire

The nurse is suctioning a client's tracheostomy when the tracheostomy becomes dislodged and the nurse is unable to replace it easily. What is the nurse's most appropriate response? Page the respiratory therapist STAT. Maintain the client's oxygenation and alert the health care provider immediately. Cover the tracheostomy stoma and apply oxygen by nasal cannula Assess the client's respiratory status and check vital signs every 1 minute for the next hour.

Maintain the client's oxygenation and alert the health care provider immediately. Explanation: If the tracheostomy becomes dislodged and is not easily replaced, the nurse should notify the primary care provider immediately, cover the tracheostomy stoma, and assess client's respiratory status.

A nursing student is preparing to administer morning care to a patient. What is the MOST important question that the nursing student should ask the patient about personal hygiene?

May I help you with a bed bath now or later this morning?

A school nurse is teaching parents about home safety and fires. What information would be accurate to include in the teaching plan? Select all that apply. 60% of U.S. fire deaths occur in the home. Most fatal fires occur when people are cooking. Most people who die in fires die of smoke inhalation. Over 1/3 of fire deaths occur in a home without a smoke detector. Fires are more likely to occur in homes without electricity or gas. More fires occur in homes occupied by single parents.

Most people who die in fires die of smoke inhalation., Over 1/3 of fire deaths occur in a home without a smoke detector., Fires are more likely to occur in homes without electricity or gas. Rationale: 85% of fires occur at home (60% too low), Most people who die of smoke inhalation die because they are sleeping not cooking, more smoke inhalation > burns. People without electricity or gas may have wood stoves, or fireplaces might be the only source of heat and they leave it on.

A nurse is discussing the benefits of smoking cessation with a client. The nurse informs the client that smoking cessation will reduce the client's risk for cancer, improve respiratory status, and enhance the quality of life. The nurse also shares a personal story of smoking cessation, provides information on other individuals who have successfully quit, and encourages the client to attend a support group for smoking cessation. The client discusses feelings on smoking cessation and verbalizes a desire to quit smoking. What type of counseling did the nurse provide to this client?

Motivational

When a client says, "I don't care if I get better; I have nothing to live for, anyway," which type of counseling would be appropriate?

Motivational counseling

A nurse manager is changing the policy for scheduling staff on a critical care unit. The schedules will be changed and an announcement was made regarding this to the staff. Based on Lewin's change theory, in what stage of change is the manager participating?

Moving

A nurse enters a patient's room and examines the patient's IV fluids and cardiac monitor. The patient states, "Well I haven't seen you before. Who are you?" What is the nurse's BEST response?

My name is John Smith, I am your nurse and I'll be caring for you until 11 pm.

The health care provider has prescribed an oropharyngeal airway for a client with a decreased level of consciousness. The health care provider has noted gurgling respirations and the client's tongue is in the posterior pharynx. The client vomits as the airway is inserted. Which actions should the nurse take? Select all that apply. Position client onto the side immediately. Remove oropharyngeal airway. Provide oral suctioning and mouth care. Raise the head of the bed to 90 degrees. Assess for bleeding in the mouth.

Position client onto the side immediately. Remove oropharyngeal airway. Provide oral suctioning and mouth care. Explanation: The nurse should quickly position client into a lateral position to prevent aspiration, remove the oropharyngeal airway, and then suction or provide oral hygiene as needed. Raising the head of bed to 90 degrees is unnecessary, because the client should be positioned on one side. There is no indication that trauma to the mouth has occurred, so the nurse would not need to assess for bleeding.

A client who recently underwent a coronary artery bypass graft is taking furosemide and metoprolol following the procedure. While developing a plan for a heart-healthy diet with the nurse, the client states that diet did not contribute to the heart disease and that the client should be fine just continuing to take the medications. According to the Transtheoretical Model of Change, which stage of change is the client in related to diet?

Precontemplation

An older adult client has developed diabetic neuropathy. What would be the most important education intervention for the client and family? Obtain a carbon monoxide detector in the home Reduce the temperature on the water heater. Keep the environment warmer in winter. Increase the amount of ventilation in the house.

Reduce the temperature on the water heater. Rationale: -Safe environments for older adults include comfortable temp range, adequate clothing, bath water on right temperature, ventilation, lighting. Neuropathy is = damage to the peripheral nerves = having trouble sensing things

A client vomits as a nurse is inserting his oropharyngeal airway. What would be the most appropriate intervention in this situation? Remove the airway, turn the client to the side, and provide mouth suction, if necessary. Immediately remove the airway, rinse the client's mouth with sterile water, and report this to the health care provider. Leave the airway in place and promptly notify the health care provider for further instructions. Suction the client's mouth through the oropharyngeal airway to prevent aspiration.

Remove the airway, turn the client to the side, and provide mouth suction, if necessary. Explanation: If the client vomits as the oropharyngeal airway is inserted, quickly position the client onto his or her side to prevent aspiration, remove the oral airway, and suction the mouth, if needed. It would be inappropriate and unsafe to leave the airway in place. Rinsing the client's mouth with water is not a priority.

A medical-surgical nurse is assisting a wound care nurse with the debridement of a client's coccyx wound. What is the primary goal of this action?

Removing dead or infected tissue to promote wound healing

What is an accurate guideline for the use of PPE?

Replace gloves if they are visibly soiled.

The nurse is preparing a sterile field for a dressing change. How would the nurse add paper-wrapped sterile items to the sterile field?

Separate the sealed flaps and drop contents onto field.

A nurse may attempt to help a client solve a situational crisis during what type of counseling session?

Short-term counseling

A nurse is assessing a client who was exposed to botulism from contaminated food supplies. Which symptom would the nurse expect to find in this client? Skin lesion with local edema that progresses, enlarges, ulcerates, and becomes necrotic Flu-like symptoms Skeletal muscle paralysis that progresses symmetrically and in a descending manner Petechial hemorrhages

Skeletal muscle paralysis that progresses symmetrically and in a descending manner Rationale: Exposure to botulism results in skeletal muscle paralysis that progresses symmetrically and in a descending manner. Muscle weakness and respiratory failure are s/s. Smallpox = flu-like s/s Skin lesions with local edema that worsens = anthrax Petechial hemorrhages = Viral hemorrhagic fevers

Which of these statements reflects the expected functioning at a hospital that has achieved Magnet status?

Staff nurses are developing innovative solutions to problems.

Stages of a wound

Stage I- inact skin with non blanchable redness Stage II- partial tissue loss such as a blister Stage III- wound with full thickness tissue loss. Stage IV- full tissue loss with exposed bone, tendon, or muscle.

A nurse is assessing a pressure ulcer on a client's coccyx area. The wound size is 2 cm × 5 cm. Approximately 30% of the wound bed is covered in yellow slough. There is an area of undermining to the right side of the wound 2 cm deep. Subcutaneous fat is visible. Which of the following stages should the nurse assign to this client's wound?

Stage III

The nurse begins a shift and finds that the wrong medication has been administered to a client. After completing a safety event report, what should the nurse do next? File the safety event report in the appropriate file and document in the nurse's notes the date and time that it was filed. Make a copy of the safety event report for the client. Place the safety event report in the client's medical record for future reference. Submit the safety report to the appropriate department within the facility so that it can be reviewed.

Submit the safety report to the appropriate department within the facility so that it can be reviewed. Rationale: -Don't give incident reports to clients or chart it, give it to department in order for the institution to learn from the experience.

When caring for a client with a tracheostomy, the nurse would perform which recommended action? Clean the wound around the tube and inner cannula at least every 24 hours. Assess a newly inserted tracheostomy every 3 to 4 hours. Use gauze dressings over the tracheostomy that are filled with cotton. Suction the tracheostomy tube using sterile technique.

Suction the tracheostomy tube using sterile technique. Explanation: Sterile technique is required when suctioning a tracheostomy in order to prevent introduction of microorganisms into the respiratory tract. The area around a new tracheostomy may need to be assessed and cleaned every 1 to 2 hours. Gauze dressings that are not filled with cotton must be used to prevent aspiration of lint or cotton fibers into the trachea.

A client has tested positive for methicillin-resistant Staphylococcus aureus after being swabbed on admission to the hospital. Bacterial resistance to antibiotics is an example of:

Survival adaptation

A nurse is caring for a client in a wound care clinic. The client has a wound on the right heel that is 2 cm × 4 cm. The wound is a maroon color and looks like a blood-filled blister. Which of the following stages should the nurse document for this wound?

Suspected deep tissure ingury. A marroon filled bilstered is often boggy or painful area.

The nurse is caring for an 80-year-old patient who was admitted to the hospital in a confused and dehydrated state. After the patient got out of bed and fell, restraints were applied. She began to fight and was rapidly becoming exhausted. She has black-and-blue marks on her wrists from the restraints. What would be the most appropriate nursing intervention for this patient? Sedate her with sleeping pills and leave the restraints on. Take the restraints off, stay with her, and talk gently to her. Leave the restraints on and talk with her, explaining that she must calm down. Talk with the client's family about taking her home because she is out of control.

Take the restraints off, stay with her, and talk gently to her. Physical restraints increase the possibility of the occurrence of falls, skin breakdown and contractures, incontinence, depression, delirium, anxiety, aspiration respiratory difficulties, and even death. The best action in this situation is for the nurse to remove the restraint, stay with the patient and gently talk to her. Sleeping pills = chemical restraint

A client is reluctant to learn to do finger sticks for home international normalized ratio (INR) monitoring. What is the best statement by the nurse?

Tell me what you know about these tests.

The nurse is advising a client about health problems related to being overweight. The client acknowledges the need to lose weight. The nurse provides education on how to count calories in food and the importance of daily exercise. The client states an intention to begin counting calories with the next meal. The client also states an intention to begin an exercise program with friends. Which statement associated with this scenario illustrates Lewin's stage of unfreezing?

The client acknowledges the need to lose weight.

When establishing a teaching-learning relationship with a client, it is most important for the nurse to remember that effective learning can best be achieved through which concept?

The client and the nurse are equal participants.

The nurse is reassessing a client after pain medication has been administered to manage the pain from a bilateral knee replacement procedure. Which statement most accurately depicts proper documentation of pain assessment?

The client reports that on a scale of 0 to 10, the current pain is a 3.

The following statement is documented in a client's health record: "Patient c/o severe H/A upon arising this morning." Which interpretation of this statement is most accurate?

The client reports waking up this morning with a severe headache.

The nurse is providing instructions to a client about performance of breast self-examination. What learning outcome would be most appropriate regarding this education?

The client will be able to perform proper breast self-examination for breast cancer detection and prevention.

What is the most appropriate outcome for the client who has a nursing diagnosis of "Risk for Injury related to the use of assistive mobility devices in an unfamiliar environment?" The client will demonstrate safety measures to prevent falls. The client will establish safety priorities with family members. The client will identify resources for safety information. The client will identify unsafe situations in his or her environment.

The client will demonstrate safety measures to prevent falls. Rationale: -Establishing safety priorities with family members CAN help, but it isn't the most appropriate outcome for the client right now. Identifying resources isn't going to fix the patient's nursing dx. -The nurse should determine that the client is at risk for falls = a good outcome would be to prevent falls.

The acute care nurse is caring for a client who is at risk for falling. Which desired outcome is most appropriate for this client?

The client will not experience a fall and remains free of injury. Rationale: Consider various factors and the environment that affect the client's safety and formulate expected outcomes for each situation. Some expected outcomes for clients that promote safety and prevent fall include: -Demonstrate safety measures to prevent falls -Remain free of injury during hospitalization

A client who is enrolled in Medicare and who has been recovering in the hospital from a stroke has developed a pressure injury on his coccyx, an event that the Centers for Medicare & Medicaid Services (CMS) has identified as a "never event." The nurse should recognize what implication of this CMS designation? The hospital must bear any costs incurred for treating the client's injury. The hospital will be fined by CMS because the client developed a pressure injury. CMS will bear the hospital's costs if the client chooses to sue the hospital. CMS may choose to divert clients to other health care facilities in the future.

The hospital must bear any costs incurred for treating the client's injury. Rationale: Never events = cost of care will NOT be paid by CMS, but by the hospital.

The nurse is teaching a new nurse about preparing a sterile field. Which action made by the new nurse would indicate further teaching is required?

The new nurse touches 1.5 in. (4 cm) from the outer edges.

A nurse is filing a safety event report for an older adult client who tripped and fell when getting out of bed. Which action exemplifies an accurate step of this process? The nurse adds the information in the safety event report to the client health record. The nurse calls the primary health care provider to fill out and sign the safety event report. The nurse provides an opinion of the physical and mental condition of the client that may have precipitated the incident. The nurse details the client's response and the examination and treatment of the client after the incident.

The nurse details the client's response and the examination and treatment of the client after the incident. Rationale: Unintentional injury that compromises safety in a health care agency needs a safety event report (incident report). Complete this after the incident and is responsible for recording it and the effect on the client in the medical record. The safety event report is NOT a part of the medical record and should not be mentioned in the documentation.

Which finding from a nursing audit reflects high standards for client safety and institutional health care?

The nurse documents clients' responses to nursing interventions.

14. The Joint Commission issues guidelines regarding the use of restraints. In which case is a restraint properly used? The nurse positions a patient in a supine position prior to applying wrist restraints. The nurse ensures that two fingers can be inserted between the restraint and patient's ankle. The nurse applies a cloth restraint to the left hand of a patient with an IV catheter in the right wrist. The nurse ties an elbow restraint to the raised side rail of a patient's bed.

The nurse ensures that two fingers can be inserted between the restraint and patient's ankle. Rationale: The patient should not be put in a supine position with restraints due to risk of aspiration. IV on right wrist, alternative forms of restraints should be tried like cloth mitt or elbow restraint.

A nurse who is caring for newborn infants delivers care by using the sense that is most highly developed at birth. What example of nursing care achieves this goal?

The nurse gently strokes the baby's cheek to facilitate breastfeeding.

A nurse enters the room of a patient with cancer. The patient is crying and states, "I feel so alone."

The nurse hold the patient's hand and asks, "What you makes you feel so alone?

The nurse is finding it difficult to plan and implement care for a client and decides to have a nursing care conference. What action would the nurse take to facilitate this process?

The nurse meets with nurses or other health care professionals to discuss some aspect of client care.

A nurse is preparing to file a safety event report after a client experienced a fall. The nurse is aware that which statement below is correct regarding the filing of a safety event report?

The nurse should record the incident in the client's medical record and fill out a safety event report separately.

A client's surgical wound dehisced when a nurse removed the staples before a health care provider order was given. Following root cause analysis, which organizational response is appropriate? Select all that apply. The nurse's actions will be deemed intentionally reckless. The nurse will be found to have committed a human error. Systems around the documentation of orders will be reviewed. The nurse will be disciplined by an impartial review board. The nurse will be sued by the hospital for malpractice

The nurse will be found to have committed a human error. Systems around the documentation of orders will be reviewed. Rationale: No malice (intentional recklessness) or failing to realize the gravity of an action (at risk behavior). Systemic factors must be examined.

A 3 year old child is being admitted to a medical division for vomiting, diarrhea, and dehydrations. During the admission interview, the nurse should implement which communication techniques to elicit the most information from the parents?

The use of clarifying questions.

The nursing instructor is discussing IV fluid overload with the nursing students. What will the nurse include in her discussion?

The use of packed cells instead of whole blood will decrease the fluid volume delivered to the client. A symptom of fluid overload is distended neck veins. Fluid overload is more likely in very young children. The infusion rate must be carefully monitored during the admin of blood.

A student nurse is preparing to perform a dressing change for a pressure ulcer on a client's sacrum area. The chart states that the pressure ulcer is staged as unstageable. Which of the following wound descriptions should the student nurse expect to assess?

The wound is 3cm x 5cm with yellow tissue covering the entire wound

A nurse who "unblocks" and "clears" congested areas of energy in a client's body to promote comfort is applying the phenomenon known as:

Therapeutic touch.

Which statement most accurately describes the process of osmosis?

Water moves from an area of lower solute concentration to an area of higher solute concentration.

A nurse is volunteering at a day camp. A child is stung by a bee and develops wheezing in the upper airways. The child is experiencing: a bronchospasm. bronchitis. bronchiectasis. bronchiolitis.

a bronchospasm. Explanation: When allergic responses take place in the lungs, breathing difficulties are far more severe. Small airways become edematous, mucus production increases, and inflammatory chemical mediators cause bronchospasm.

The nurse sets up an oxygen tent for a client. Which client is the best candidate for this oxygen delivery system? an older adult client who has COPD a child who has pneumonia an adult who is receiving oxygen at home an adolescent who has asthma

a child who has pneumonia Explanation: An oxygen tent is commonly used with children who need a cool and highly humidified airflow. It is also more effective for children because they often do not like to keep oxygen administration devices in place. Since the tent does not allow the maintenance of a satisfactory or precise oxygen concentration, is difficult to maintain a consistent level of oxygen. The oxygen tent does not adequately deliver oxygen at a rate higher than 30% to 50%; thus, it is rarely used with other clients.

Which documentation tool will the nurse use to record the client's vital signs every 4 hours?

a flow sheet - Explanation: A flow sheet is a form used to record specific client variables such as pulse, respiratory rate, blood pressure readings, body temperature, weight, fluid intake and output, bowel movements, and other client characteristics.

The oncoming nurse is assigned to the following clients. Which client should the nurse assess first?

a newly admitted 88-year-old with a 2-day history of vomiting and loose stools

The nurse is assessing the wounds of patients in a burn unit. Which wound would most likely heal by primary intention?

a surgical incision with sutured approximated edges.

The nurse is preparing a client for a cystoscopy procedure. Which intervention would be part of the preparation for this? a) Having the client sign a consent form for the procedure b) Explaining to the client that the procedure will be painful c) Maintaining the client without liquids before the procedure d) Inserting a Foley catheter the morning of the procedure

a) Having the client sign a consent form for the procedure

the nurse is selecting antidiarrheal medications for patients with diarrhea. Which statements accuraely describe the action of specific antidiarrheal medications? (SATA) a) Diphenoxylate should not be used if antibiotic-associated diarrhea is suspected b) pepto-bismol contains salicylates; a health care provider should be consulted before giving it to children or patient taking aspirin c) higher than recommended doses of loperamide may increase serious cardiac events d) laperamide has a shorter duration than diphenoxylate and atropine

a, b, c

A nurse is collecting a stool specimen from a patient. Which measures are appropriate for this procedure? (SATA) a) the patient should be asked to void first because the lab study may be inaccurate if the stool contains urine b) the patient should be instructed not to place toilet tissue in the bedpan or specimen container c) medical aseptic techniques are always followed d) handwashing is performed before and after glove use when handling a stool specimen e) generally, 2 in of formed stool or 20 to 30 mL of liquid stool is sufficient for a stool specimen

a, b, c, d

The nurse is preparing to insert an indwelling urinary catheter into a female client's bladder. Arrange the following steps in the correct order. a) Open sterile catheterization tray using sterile technique. b) Don sterile gloves. c) Open all sterile supplies. d) Clean each labial fold, then the area directly over the meatus. e) Slowly insert the lubricated catheter into the urethra. f) Advance the catheter until there is a return of urine.

a, b, c, d, e, f

A nurse who is newly hired to manage a busy pediatric office is encouraged to use a transactional leadership style when dealing with subordinates. Which activities best exemplify the use of this type of leadership? Select all that apply. a.The manager institutes a reward program for employees who meet goals and work deadlines. b.The manager encourages the other nurses to participate in health care reform by joining nursing organizations. c.The manager promotes compliance by reminding subordinates that they have a good salary and working conditions. d.The manager makes sure all the employees are kept abreast of new developments in pediatric nursing. e.The manager works with subordinates to accomplish all the nursing tasks and goals for the day. f.The manager allows the other nurses to set their own schedules and perform nursing care as they see fit.

a, c

Which of the following commonly used enema solutions would the nurse administer to distend the intestine and increase peristalsis? (SATA) a) tap water b) soap c) normal saline d) hypertonic

a, c

A nurse is assessing the bowel elimination patterns of hospitalized patients. Which nursing actions related to the assessment process are performed correctly? (SATA) a) the nurse places the patient in the supine position with the abdomen exposed b) the nurse drapes the patient's chest and public area and extends the patient's legs flat against the bed c) the nurse uses a warmed stethoscope to listen for bowel sounds in all abdominal quadrants d) the nurse notes the character of bowel sounds, which are normally high pitched, gurgling, and soft

a, c, d

The nurse is performing digital removal of a fecal impaction. Which nursing actions follow guidelines for this procedure? (SATA) a) place the patient in a side-lying position b) vigorously work the finger around and into the hardened mass to break break it up c) use nonsterile gloves for the procedure because the intestinal tract is not sterile d) lubricate the index finger generously to reduce irritating the rectum, and insert the finger gently into the anal canal

a, c, d

decentralized decision-making process

autonomous, accountable professional nursing practice; a characteristic of a democratic leadership style and the heart of a self-governance model of unit organization

A charge nurse in a busy hospital manages a skilled nursing unit using an autocratic style of leadership. Which leadership tasks BEST represent this style of leadership? Select all that apply. a.The charge nurse polls the other nurses for input on nursing protocols. b.The charge nurse dictates break schedules for the other nurses. c.The charge nurse schedules a mandatory in-service training on new equipment. d.The charge nurse allows the other nurses to divide up nursing tasks. e.The charge nurse delegates nursing responsibilities to the staff. f.The charge nurse encourages the nurses to work independently.

b, c, e

A nurse is assessing the bowel elimination of patient in med-surg unit. What developmental factors affecting elimination should the nurse consider? (SATA) a) Voluntary control of defecation occurs between the ages of 12 and 18 months b) the number of stools that infants pass varies greatly c) some children have bowel movements old every 2 or 3 days d) in an infant, a liquid stool signifies diarrhea e) constipation is often a chronic problem for older adults

b, c, e

A nurse who is planning menus for a patient in a long-term care facility takes into consideration the effects of foods and fluids on bowel elimination. Which examples correctly describe these effect? (SATA) a) a patient who is constipated should eat eggs and pasta to relieve the condition b) patients who are constipated should eat more fruits and vegetables c) patients experiencing flatulence should avoid gas-producing food such as cauliflower and onions d) alcohol and coffee tend to have a constipating effect on patients e) patients with food intolerances may experience altered bowel elimination

b, c, e

The nurse is administering an oil-retention enema to a patient. Which nursing actions in this procedure are performed correctly? (SATA) a) the nurse chooses a large rectal tube b) the nurse administers the oil-retention enema at body temperature c) the nurse instructs the patient to retain the oil for at least 30 minutes d) the nurse administers a cleaning enema prior to the oil-retention enema e) the nurse administers a cleansing enema after the oil-retention enema

b, c, e

The nurse inspects the stool of patients admitted to the hospital with abdominal distress. Which statements accurately describe the normal characteristics of stool and special considerations for observation? (SATA) a) consistently large diarrheal stools suggest a disorder of the left colon or rectum b) the rapid rate of peristalsis in the breastfed infant causes the stool to be yellow c) the absence of bile may cause the stool to appear black d) antacids in the diet cause the stool to be whitish e) a gastrointestinal obstruction may result in a narrow, pencil-shaped stool

b, d, e

The nurse is changing a patient's ostomy appliance and observes that the peristomal skin is excoriated. What would be the nurse's priority intervention in this situation? a) notify the primary care provider b) suspect ischemia and notify the primary care provider immediately c) clean outside of bad thoroughly when emptying d) make sure that the appliance is not cut too large.

c

Which of the following is a recommended guideline when catheterizing the female urinary bladder? a) Lubricate 2 to 3 inches (5-7.5 cm) of the catheter tip before insertion. b) Using the dominant hand, hold the catheter by the tip and insert slowly into the urethra. c) Advance the catheter until there is a return of urine (approximately 4 to 5 inches [10-12.5 cm]) d) Once urine drains, advance the catheter another 2 to 3 inches (2.5-5 cm).

d) Once urine drains, advance the catheter another 2 to 3 inches (2.5-5 cm). d) Once urine drains, advance the catheter another 2 to 3 inches (2.5-5 cm).

A nurse manager of a busy cardiac unit observes disagreements between the RNs and the LPNs related to schedules and nursing responsibilities. At a staff meeting, the manager compliments all the nurses on a job well done and points out that expected goals and outcomes for the month have been met. The nurse concludes the meeting without addressing the disagreements between the two groups of nurses. Which conflict resolution strategy is being employed by this manager? a.Collaborating b.Competing c.Compromising d.Smoothing

d, resolves conflict by complimenting the parties involved and focusing on agreement rather than disagreement is using smoothing to reduce the emotion in the conflict

The nurse is taking care of a client on the second post-operative day who asks about wound dehiscence. Which response by the nurse is most accurate?

dehiscence is when a would has partial or total seperation of the wound layers

A nurse manager values group satisfaction and tries to motivate the staff to be the best nurses possible. In which style of leadership is the group satisfaction and motivation the primary benefit?

democratic

when the manager is leading others in achieving goals within the constraints of the current fiscal and workforce shortage scenarios they are fulfilling which role? (toughest role)

directing

The nurse is caring for a client who has been prescribed humidified oxygen at 6 L/minute. Which type of liquid will the nurse gather to set up the humidifier? tap water normal saline distilled water mineral oil

distilled water Explanation: Distilled water is used when humidification is desired. Other answers are incorrect.

The nurse is implementing an order for oxygen for a client with facial burns. Which delivery device will the nurse gather? face tent simple mask nasal cannula tracheostomy collar

face tent Explanation: A face tent is used without a mask; it is open and loose around the face and is often used for patients with facial trauma or burns. A simple mask or nasal cannula would irritate the facial skin. The client does not have a tracheostomy.

The nurse is caring for a client who was found without food or water for 2 days in the desert. What explanation for the need for fluid does the client have? Select all that apply.

facilitates cellular metabolism helps maintain normal body temperature acts as a solvent for electrolytes

A nurse on a medical unit recognizes the need to demonstrate Quality and Safety Education for Nurses (QSEN) competencies in clinical practice. Which action best demonstrates the skills necessary to meet the QSEN competency of safety? filling out an incident report accurately after a client went missing from the unit appreciating the relationship between continuing education and client safety understanding the functions of a new automated intravenous pump that has been introduced to the unit valuing the contributions of clients and their families who suggest possible improvements in care

filling out an incident report accurately after a client went missing from the unit. Rationale: "Valuing" and "appreciating" = attitude, "understanding" is knowledge. While filling out an incident report is a SKILL that aligns with the QSEN competency of safety.

The nurses on a critical care unit can utilize the safety strategy of redundancy by: having two nurses independently check the dosage of high-risk medications. ensuring the antidotes are readily available for certain high-risk medications. introducing equipment that makes it more difficult for a nurse to commit an error. introducing a brief waiting period between the time that a medication is ordered and the time that it is administered.

having two nurses independently check the dosage of high-risk medications. Rationale: Successive checks for high risk procedures or meds add needed safety redundancy. Equipment that makes it difficult to make errors is an example of mistake-proofing.

A nurse on a night shift entered an older adult client's room during a scheduled check and discovered the client on the floor beside the bed, the result of falling when trying to ambulate to the washroom. After assessing the client and assisting into the bed, the nurse has completed an incident report. What is the primary purpose of this particular type of documentation?

identifying risks and ensuring future safety for clients

To eliminate needlesticks as potential hazards to nurses, the nurse should:

immediately deposit uncapped needles into a puncture-proof plastic container.

autocratic leadership

leadership style in which the leader assumes complete control over the decisions and activities of the group

quantum leadership

leadership that moves beyond the traditional modes

When documenting the care of a client, the nurse is aware of the need to use abbreviations conscientiously and safely. This includes:

limiting abbreviations to those approved for use by the institution.

transactional leadership

maintaining control by rewarding good behavior and punishing negative or detrimental behavior

You are preparing to irrigate a patient's wound. Arrange the following steps in the correct order.

mask gown eyes take off soiled sterile gloves fill syringe direct stream dry area

do transactional leaders inspire nurses to participate?

no, transactional leaders minimize employees thinking and involvement

The nurse is caring for a client who became very ill after ingesting seafood. How will the nurse document this condition?

noncommunicable disease

A nurse is taking care of a client with tuberculosis who has developed resistance to the ordered antibiotic. Which type of client is most likely at increased risk for infection?

older adult

In which situations has the nurse used a dressing properly?

opsite aseptic techniqes sof wick

when a manager is acquiring, managing, and mobilizing resources to meet both clinical and financial objectives what role are they fulfilling?

organizing

A nurse manager reviews an employee's contribution to the nursing division annually. This process is:

performance appraisal.

transformational leadership

person creates revolutionary change and commits to the personal and professional growth of self and others

A nurse is maintaining a problem-oriented medical record for a client. Which component of the record describes the client's responses to what has been done and revisions to the initial plan?

progress notes

The nurse notices a student preparing to enter the room of a client with pulmonary tuberculosis with only gloves on. What is the appropriate nursing intervention?

remind the student that a fitted N95 respirator is required

A client with uncontrolled diabetes develops hypophosphatemia. Which finding would the nurse most likely assess? Select all that apply.

respiratory muscle weakness confusion ventricular dysrhythmia

In a centralized management structure who makes decisions?

senior managers

which situations is democratic leadership not good for?

situations where a rapid response is needed because democratic takes long

A nurse is caring for a client diagnosed with myocardial infarction. A person identifying himself as the client's friend asks the nurse for the client's records, but the nurse declines. The nurse's unwillingness to divulge the requested information is based on the understanding that which people would be entitled to access to the client's records?

those directly involved in the client's care

The nurse is caring for a client who has had a percutaneous tracheostomy (PCT) following a motor vehicle accident and has been prescribed oxygen. What delivery device will the nurse select that is most appropriate for this client? simple mask tracheostomy collar nasal cannula face tent

tracheostomy collar Explanation: A tracheostomy collar delivers oxygen near an artificial opening in the neck. This is appropriate for a client who has had a PCT. All other devices are less appropriate for this client.

A client's record can be more accurate if the nurse:

uses point-of-care documentation.

conflict management

work through conflicts in a way that minimizes negative effects and promotes positive consequences

Which actions would a nurse be expected to perform when applying a saline-moistened dressing to a patient's wound? (Select all that apply.)

• Gently press to loosely pack the moistened gauze into the wound; if necessary, use forceps or cotton-tipped applicators to press gauze into all wound surfaces. • Carefully and gently remove the soiled dressings; if there is resistance, use a silicone-based adhesive remover to help remove the tape. • Position the patient so the wound cleanser or irrigation solution will flow from the clean end of the wound toward the dirtier end.

A child is admitted to the pediatric division with an acute asthma attack. The nurse assesses the lung sounds and respiratory rate. The mother asks the nurse, "Why is his chest sucking in above his stomach? The nurse's most accurate response is: "He will require additional testing to determine the cause." "He is using his chest muscles to help him breathe." "His infection is causing him to breathe harder." "His lung muscles are swollen so he is using abdominal muscles."

"He is using his chest muscles to help him breathe." Explanation: The client will use accessory muscles to ease dyspnea and improve breathing.

The nurse working at the blood bank is speaking with potential blood donor clients. Which client statement requires nursing intervention?

"I received a blood transfusion in the United Kingdom."

A nursing instructor is discussing administration of total parenteral nutrition (TPN) with a nursing student. Which statement by the student would require further teaching?

"I will be sure to change the TPN tubing every other day."

A client admitted to the facility is diagnosed with metabolic alkalosis based on arterial blood gas values. When obtaining the client's history, which statement would the nurse interpret as a possible underlying cause?

"I've been taking antacids almost every 2 hours over the past several days." Metabolic alkalosis occurs when there is excessive loss of body acids or with unusual intake of alkaline substances. It can also occur in conjunction with an ECF deficit or potassium deficit. V or nasogastric suction diarrhea = metabolic acidosis

The nurse is demonstrating oxygen administration to a client. Which teaching will the nurse include about the humidifier? "This is a gauge used to regulate the amount of oxygen that a client receives." "The humidifier prescribes the concentration of oxygen." "It measures the percentage of delivered oxygen to determine whether the client is getting the amount prescribed." "Small water droplets come from this, thus preventing dry mucous membranes."

"Small water droplets come from this, thus preventing dry mucous membranes." Explanation: The humidifier produces small water droplets which are delivered during oxygen administration to prevent or decrease dry mucous membranes. The oxygen analyzer measures the percentage of delivered oxygen to determine whether the client is receiving the prescribed amount. The flowmeter is a gauge used to regulate the amount of oxygen that a client receives. The healthcare provider prescribed the concentration of oxygen.

The nursing instructor is quizzing a group of students about fluid and electrolyte balance. Which statements made by students indicate an understanding of the efforts of the organs to maintain fluid and electrolyte balance? Select all that apply.

"The kidneys regulate extracellular fluid volume by retention and excretion of body fluids." "The kidneys regulate pH of extracellular fluid by excreting and retaining hydrogen ions." The adrenal glands regulate blood volume by secreting aldosterone." "The nervous system regulates oral intake by sensing intracellular dehydration, which in turn stimulates thirst." The heart and blood vessels react to hypovolemia by stimulating fluid retention rather than the kidneys.

A client who is receiving total parenteral nutrition and lipids asks the nurse why the solution looks like milk. What is the most appropriate nursing response?

"The white milky solution contains lipids or fat to provide extra calories." A parenteral lipid emulsion is a mixture of water and fats in the form of soybean or safflower oil, egg yolk phospholipids, and glycerin. Lipid solutions, which look milky white, are given intermittently with TPN solutions. They provide additional calories and promote adequate blood levels of fatty acids. Lipids cannot be mixed with TPN, as the lipid molecules tend to break or separate

A client who uses portable home oxygen states, "I still like to smoke cigarettes every now and then." What is the appropriate nursing response? "An occasional cigarette will not hurt you." "You should never smoke when oxygen is in use." "I understand; I used to be a smoker also." "Oxygen is a flammable gas."

"You should never smoke when oxygen is in use." Explanation: The nurse will educate the client about the dangers of smoking when oxygen is in use. Oxygen is not flammable, but it oxidizes other materials. Other answers are inappropriate.

A healthy client eats a regular, balanced diet and drinks 3,000 mL of liquids during a 24-hour period. In evaluating this client's urine output for the same 24-hour period, the nurse realizes that it should total approximately how many mL?

3,000 Fluid intake and fluid output should be approximately the same in order to maintain fluid balance. Any other amount could lead to a fluid volume excess or deficit

The nurse is calculating the infusion rate for the following order: Infuse 1000 ml of 0.9% NaCl over 8 hours, with gravity infusion. Your tubing delivers 20 gtts/min. What is the infusion rate?

42 gtts/min When infusing by gravity, divide the total volume in ml (1000 ml) by the total time in minutes (480 minutes) times the drop factor, which is given as 20 gtts/min

A physician orders an infusion of 250 mL of NS in 100 minutes. The set is 20 gtt/mL. What is the flow rate?

50 gtt/min The flow rate (gtt/min) equals the volume (mL) times the drop factor (gtt/mL) divided by the time in minutes.

The nurse works at an agency that automatically places certain clients on intake and output (I&O). For which client will the nurse document all I&O?

55-year old with congestive heart failure on furosemide clients who have undergone surgery until they are eating, drinking, and voiding in sufficient quantities; those on IV fluids or receiving tube feedings; those with wound drainage or suction equipment; those with urinary catheters; and those on diuretic drug therapy should be put on I&O

The nurse is calculating an infusion rate for the following order: Infuse 1000 ml of 0.9% Na Cl over 12 hours using an electronic infusion device. What is the infusion rate?

83 ml/hour When calculating the infusion rate with an electronic device, divide the total volume to be infused (1000 ml) by the total amount of time in hours (12). This is 83 ml/hour

A nurse assessing a client's respiratory effort notes that the client's breaths are shallow and 8 per minute. Shortly after, the client's respirations cease. Which form of oxygen delivery should the nurse use for this client? Oxygen mask Nasal cannula Ambu bag Oxygen tent

Ambu bag Explanation: If the client is not breathing with an adequate rate and depth, or if the client has lost the respiratory drive, a manual resuscitation bag (Ambu bag) may be used to deliver oxygen until the client is resuscitated or can be intubated with an endotracheal tube. Oxygen masks may cover only the nose and mouth and can vary in the amount of oxygen delivered. A nasal oxygen cannula is a device that consists of a plastic tube that fits behind the ears, and a set of two prongs that are placed in the nostril. An oxygen tent is a tentlike enclosure within which the air supply can be enriched with oxygen to aid a client's breathing. Oxygen masks, nasal cannula, and oxygen tents are used for clients who have a respiratory drive.

What is the lab test commonly used in the assessment and treatment of acid-base balance?

Arterial Blood Gas

The nurse is educating an adolescent with asthma on how to use a metered-dose inhaler. Which education point follows recommended guidelines? Inhale through the nose instead of the mouth. Be sure to shake the canister before using it. Inhale the medication rapidly. Inhale two sprays with one breath for faster action.

Be sure to shake the canister before using it. Explanation: A metered-dose inhaler (MDI) delivers a controlled dose of medication with each compression of the canister. The canister must be shaken to mix the medication properly. MDIs are inhaled through the mouth, into the lungs. The medication should be inhaled slowly to ensure a sufficient dose enters the lungs. If the order is for two sprays, these sprays are administered with one spray for each breath. The inhaled breath should be held briefly after each spray in order to prevent immediately exhaling the medication.

The student nurse asks the instructor how buffer systems work in the body to maintain the pH of the blood. The instructor explains each of the buffer systems to the students. Which are buffer systems that will be discussed by the instructor?

Carbonic acid-sodium bicarbonate buffer system. Phosphate buffer system. Protein buffer system.

The nurse is caring for elderly patients in a long-term care facility. What age-related alteration should the nurse consider when planning care for these patients?

Cardiac volume intolerance

Which clients would be appropriate candidates for total parenteral nutrition? Select all that apply.

Client who has second- and third-degree (partial- or full-thickness) burns over 40% of the body Client who had gastric surgery and is unable to eat for a few weeks Client with anorexia nervosa

A client is taking a diuretic such as furosemide. When implementing client education, what information should be included?

Decreased potassium levels Many diuretics such as furosemide are potassium wasting; hence, potassium levels are measured to detect hypokalemia.

The nurse reviews the laboratory test results of a client and notes that the client's potassium level is elevated. What would the nurse expect to find when assessing the client's gastrointestinal system?

Diarrhea Abdominal distention, vomiting, and paralytic ileus would reflect hypokalemia.

The nurse is providing care to a client who has a serum potassium level of 5.2 mEq/L (5.2 mmol/L). Which findings would the nurse expect to assess? Select all that apply.

Diarrhea Cardiac dysrhythmia

Which dietary guideline would be appropriate for the older adult homebound client with advanced respiratory disease who informs the nurse that she has no energy to eat? Snack on high-carbohydrate foods frequently. Eat smaller meals that are high in protein. Contact the physician for nutrition shake. Eat one large meal at noon.

Eat smaller meals that are high in protein. Explanation: The client should consume a diet in which the body can produce plasma proteins. The client should have sufficient caloric and protein intake for respiratory muscle strength.

Which of the following statements is an appropriate nursing diagnosis for an client 80 years of age diagnosed with congestive heart failure, with symptoms of edema, orthopnea, and confusion?

Extracellular Volume Excess related to heart failure, as evidenced by edema and orthopnea

A nurse is providing care to a client with hypocalcemia. The nurse would monitor the client's laboratory test results for which imbalance?

Hyperphosphatemia Calcium and phosphorus have a reciprocal relationship

The nurse is caring for a client, who was admitted after falling from a ladder. The client has a brain injury which is causing the pressure inside the skull to increase, which may result in a lack of circulation and possible death to brain cells. Considering this information, which intravenous solution would be most appropriate?

Hypertonic Plasma is an isotonic solution.

A client is admitted to the facility after experiencing uncontrolled diarrhea for the past several days. The client is exhibiting signs of a fluid volume deficit. When reviewing the client's laboratory test results, which electrolyte imbalance would the nurse most likely find?

Hypokalemia Intestinal secretions contain bicarbonate. For this reason, diarrhea may result in metabolic acidosis due to depletion of base. Intestinal contents also are rich in sodium, chloride, water, and potassium, possibly contributing to an ECF volume deficit and hypokalemia

The nurse is caring for a client with metabolic alkalosis whose breathing rate is 8 breaths per minute. Which arterial blood gas data does the nurse anticipate finding?

In metabolic acidosis, arterial blood gas results are anticipated to reflect pH guch as 64; 42.

Which solution is a crystalloid solution that has the same osmotic pressure as that found within the cells of the body and is used to expand the intravascular volume?

Isotonic

To determine the quality of oxygenation, the nurse performs the physical assessment, the arterial blood gas test, and pulse oximetry. What is the purpose of the pulse oximetry test? Monitor the pressure of oxygen dissolved in plasma. Measure the volume of air exhaled or inhaled over time. Calculate the pressure of carbon dioxide dissolved in plasma. Monitor the amount of oxygen saturation in the blood.

Monitor the amount of oxygen saturation in the blood. Explanation: The pulse oximetry test is a noninvasive transcutaneous technique for periodically or continuously monitoring the oxygen saturation of blood. The arterial blood gases test the client's blood for the partial pressure of oxygen dissolved in plasma, the percentage of hemoglobin saturated with oxygen, and the partial pressure of carbon dioxide dissolved in plasma. Spirometry measures the volume of air in liters exhaled or inhaled by a client over time.

When educating a client about foods that affect fluid balance, the nurse would advise the client to decrease:

Na+ Sodium (Na+) is the most abundant electrolyte in the extracellular fluid (ECF). Na+ regulates extracellular fluid volume; Na+ loss or gain is accompanied by a loss or gain of water. Potassium (K+) is the major intracellular electrolyte. Calcium (Ca++) is a major component of bones and teeth. Magnesium (Mg++) is the most abundant intracellular cation after potassium.

A client with a history of chronic obstructive pulmonary disease (COPD) has been ordered oxygen at 3 L/min as needed for treatment of dyspnea. What delivery mode is most appropriate to this client's needs? Nasal cannula Simple mask Partial rebreather mask Nonrebreather mask

Nasal cannula Explanation: A nasal cannula is used to deliver from 1 L/min to 6 L/min of oxygen. Masks are used with higher flow rates of oxygen.

Potassium is essential for normal cardiac, neural, and muscle function and contractility of all muscles. Which is false about potassium?

Normal serum potassium ranges from 5.5 to 6.0 mEq/L (5.5 to 6.0 mmol/L). Normal serum potassium ranges from 3.5 to 5.0 mEq/L (3.5 to 5.0 mmol/L).

A client who recently had surgery is bleeding. What blood product does the nurse anticipate administering for this client?

Platelets

A client's primary care provider has informed the nurse that the client will require thoracentesis. The nurse should suspect that the client has developed which disorder of lung function? Pleural effusion Tachypnea Wheezes Pneumonia

Pleural effusion Explanation: Thoracentesis involves the removal of fluid from the pleural space, either for diagnostic purposes or to remove an accumulation of fluid in this space (pleural effusion). Tachypnea and wheezes are not symptoms that directly indicate a need for thoracentesis. Pneumonia would necessitate the procedure only if the infection resulted in pleural effusion.

A 50-year-old client with hypertension is being treated with a diuretic. The client reports muscle weakness and falls easily. The nurse should assess which electrolyte?

Potassium Diuretics, commonly given to treat high blood pressure and heart failure, can cause an extracellular deficit or loss of electrolytes including potassium, calcium, and magnesium.

A client is admitted to the intensive care unit with a calcium level of 4.2 mg/dL. What is the priority action by the nurse?

Prepare to administer calcium gluconate as prescribed

When reviewing data collection on a client with a cardiac output of 2.5 L/minute, the nurse inspects the client for which symptom? Rapid respirations Weight loss Increased urine output Mental alertness

Rapid respirations Explanation: Normal cardiac output averages from 3.5 L/minute to 8.0 L/minute. With decreased cardiac output, there is a reduction in the amount of circulating blood that is available to deliver oxygen to the tissues. The body compensates by increasing respiratory rate to increase oxygen delivery to the tissues. The client with decreased cardiac output would gain weight, have decreased urine output, and display mental confusion.

A client has been receiving intravenous (IV) fluids that contain potassium. The IV site is red and there is a red streak along the vein that is painful to the client. What is the priority nursing action

Remove the IV. likely has phlebitis, which is caused by prolonged use of the same vein or irritating fluid.

The nurse schedules a pulmonary function test to measure the amount of air left in a client's lungs at maximal expiration. What test does the nurse order? Tidal volume (TV) Total lung capacity (TLC) Forced Expiratory Volume (FEV) Residual Volume (RV)

Residual Volume (RV) Explanation: During a pulmonary function test the amount of air left in the lungs at the end of maximal expiration is called residual volume. Tidal volume refers to the total amount of air inhaled and exhaled with one breath. Total lung capacity is the amount of air contained within the lungs at maximum inspiration. Forced expiratory volume measures the amount of air exhaled in the first second after a full inspiration; it can also be measured at 2 or 3 seconds.

The nurse is caring for a postoperative client who has a prescription for meperidine 75 mg intramuscularly (IM) every 4 hours as needed for pain. Before and after administering meperidine, the nurse would assess which most important sign? Apical pulse Orthostatic blood pressure Respiratory rate and depth Urinary intake and output

Respiratory rate and depth Explanation: The client receiving narcotics/opioids needs monitoring of the respiratory rate and depth to ensure that respiratory depression does not result in progressive respiratory issues, physiologic damage from respiratory depression, or loss of consciousness. The pulse, blood pressure, and urinary intake and output are not as important as respiratory status when administering narcotics.

Sodium is the most abundant cation in the extracellular fluid. Which is true regarding sodium?

Sodium is regulated by the renin-angiotensin-aldosterone system.

The nurse is educating a client about the function of sodium in the body. What education points would the nurse make? Select all that apply.

Sodium is the primary regulator of ECF volume Sodium is normally maintained in the body within a relatively narrow range, and deviations quickly result in serious health problems. Sodium participates in the generation and transmission of nerve impulses.

The nurse is caring for a client with shortness of breath who is receiving oxygen at 4 L/minute. Which assessment finding will demonstrate that oxygen therapy is effective? clubbing of fingers respirations 26 breaths/minute heart rate 110 beats/minute SpO2 92%

SpO2 92% Explanation: An SpO2 at or above 90% is normal, reflecting that oxygen therapy is being effective. Clubbing, respirations greater than 26 breaths/minute, and a heart rate greater than 100 beat per minute may indicate that more oxygen is needed.

A client reports she has lactose intolerance and questions the nurse about alternative sources of calcium. What options can be provided by the nurse?

Spinach Sardines, whole grains, and green leafy vegetables also provide calcium.

The nurse is auscultating the lungs of a client and detects normal vesicular breath sounds. What is a characteristic of vesicular breath sounds? They are loud, high-pitched sounds heard primarily over the trachea and larynx. They are medium-pitched blowing sounds heard over the major bronchi. They are low-pitched, soft sounds heard over peripheral lung fields. They are soft, high-pitched discontinuous (intermittent) popping lung sounds.

They are low-pitched, soft sounds heard over peripheral lung fields. Explanation: Normal breath sounds include vesicular (low-pitched, soft sounds heard over peripheral lung fields), bronchial (loud, high-pitched sounds heard primarily over the trachea and larynx), and bronchovesicular (medium-pitched blowing sounds heard over the major bronchi) sounds. Crackles are soft, high-pitched discontinuous (intermittent) popping sounds.

A nurse needs to select a venipuncture site to administer a prescribed amount of IV fluid to a client. THe nurse looks for a large vein when using a needle with a large guage. What explains the nurse's action?

To prevent cocmpromising circulation.

When the nurse is starting an intravenous infusion on a client who will be receiving multiple intravenous antibiotics, which guideline should the nurse follow?

Use distal veins before proximal veins leaving the more proximal sites for later venipunctures.

Which guideline is recommended for determining suction catheter depth when suctioning an endotracheal tube? Combine the length of the endotracheal tube and any adapter being used, and add an additional 2 cm. Using a suction catheter with centimeter increments on it, insert the suction catheter into the endotracheal tube until the centimeter markings on both the endotracheal tube and catheter align, and insert the suction catheter no further than an additional 1 cm. Using a spare endotracheal tube of the same size as being used for the client, insert the suction catheter halfway to the end of the tube and note the length of catheter used to reach this point. For a closed system, combine the length of the endotracheal or tracheostomy tube and any adapter being used, and add an additional 3 cm.

Using a suction catheter with centimeter increments on it, insert the suction catheter into the endotracheal tube until the centimeter markings on both the endotracheal tube and catheter align, and insert the suction catheter no further than an additional 1 cm. Explanation: Guidelines to determine suction catheter depth include the following: Using a suction catheter with centimeter increments on it, insert the suction catheter into the endotracheal tube until the centimeter markings on both the endotracheal tube and catheter align, and insert the suction catheter no further than an additional 1 cm past the length of the endotracheal tube. Combine the length of the endotracheal tube and any adapter being used, and add an additional 1 cm. Using a spare endotracheal or tracheostomy tube of the same size as being used for the client, insert the suction catheter to the end of the tube and note the length of catheter used to reach the end of the tube. For a closed system, combine the length of the endotracheal or tracheostomy tube and any adapter being used, and add an additional 1 cm.

In which client would the nurse assess for a depressed respiratory system? a client taking amlodipine for hypertension a client taking antibiotics for a urinary tract infection a client taking insulin for diabetes a client taking opioids for cancer pain

a client taking opioids for cancer pain Explanation: Many medications affect the function of, and depress, the respiratory system. The nurse should monitor clients taking certain medications, such as opioids, for rate and depth of respirations. Amlodipine is a calcium channel blocker and the medication decreased blood pressure, so the nurse would need to assess blood pressure. Antibiotics are used for urinary tract infections as well as other infections and the infections do not affect the respiratory system. Insulin decreases blood sugar which a person with diabetes may need to take every day. Insulin does not affect the respiratory system.

The nurse is caring for a client who had a parathyroidectomy. Upon evaluation of the client's laboratory studies, the nurse would expect to see imbalances in which electrolytes related to the removal of the parathyroid gland?

calcium and phosphorus

Potassium is needed for neural, muscle, and:

cardiac function.

A client's most recent blood work indicates a K+ level of 7.2 mEq/L (7.2 mmol/L), a finding that constitutes hyperkalemia. For what signs and symptoms should the nurse vigilantly monitor?

cardiac irregularities

The nurse is informed while receiving a nursing report that the client has been hypoxic during the evening shift. Which assessment finding is consistent with hypoxia? confusion decreased blood pressure decreased respiratory rate hyperactivity

confusion Explanation: Anxiety, restlessness, confusion, or drowsiness are common signs of hypoxia. Hyperactivity is not associated with hypoxia. Other common symptoms of hypoxia are dyspnea, elevated blood pressure with small pulse pressure, increased respiratory and pulse rates, pallor, and cyanosis.

A client has edema of the feet and ankles, along with crackles in the lower lobes and a frothy, productive cough. The client is suffering from: pulmonary embolism. myocardial infarction. lung cancer. congestive heart failure.

congestive heart failure. Explanation: A client who has edema and a cough that is productive with frothy sputum is manifesting heart failure.

A client has been admitted with fluid volume excess related to left sided heart failure. Which assessment data would the nurse document related to the fluid volume excess? (Select all that apply.)

crackles in the lungs distended neck veins and DECREASED URINE OUTPUT

A client with no prior history of respiratory illness has been admitted to a postoperative unit following foot surgery. What intervention should the nurse prioritize in an effort to prevent postoperative pneumonia and atelectasis during this time of reduced mobility following surgery? educating the client on the use of incentive spirometry educating the client on pursed-lip breathing techniques oropharyngeal suctioning twice daily administration of inhaled corticosteroids

educating the client on the use of incentive spirometry Explanation: Incentive spirometry maximizes lung inflation and can prevent or reduce atelectasis and help mobilize secretions. Pursed-lip breathing primarily addresses dyspnea and anxiety. Suctioning is only indicated when clients are unable to independently mobilize secretions. Corticosteroids are not typically used as a preventive measure for respiratory complications after surgery.

A client loses consciousness after strenuous exercise and needs to be admitted to a health care facility. The client is diagnosed with dehydration. The nurse knows that the client needs restoration of:

electrolytes. Non-electrolytes are chemical compounds that remain bound together when dissolved in a solution. Interstitial fluid is the fluid in the tissue space between and around cells. Colloids are substances that do not dissolve into a true solution and do not pass through a semipermeable membrane.

A client has been put on oxygen therapy because of low oxygen saturation levels in the blood. What should the nurse use to regulate the amount of oxygen delivered to the client? oxygen analyzer nasal strip nasal cannula flow meter

flow meter Explanation: The nurse should use a flow meter to regulate the amount of oxygen delivered to the client. A flow meter is a gauge used to regulate the amount of oxygen delivered to the client and is attached to the source of oxygen. An oxygen analyzer is a device that measures the percentage of delivered oxygen to determine if the client is receiving the amount prescribed by the physician. An adhesive nasal strip increases the nasal diameter and promotes easier breathing. A nasal cannula is a hollow tube used for delivering a small concentration of oxygen. However, these devices are not used to regulate the amount of oxygen delivered to the client.

The nurse is caring for a client who is diagnosed with Impaired Gas Exchange. While performing a physical assessment of the client, which data is the nurse likely to find, keeping in mind the client's diagnosis? high temperature high respiratory rate low pulse rate low blood pressure

high respiratory rate Explanation: A client diagnosed with Impaired Gas Exchange has difficulty in breathing, so the nurse is likely to find a high respiratory rate. As a compensatory mechanism to impairment in gas exchange, the peripheral temperature drops, and the pulse rate and blood pressure increase.

Study blue

https://www.studyblue.com/notes/note/n/ch-38-prep-u/deck/17157979 https://www.easynotecards.com/notecard_set/67193 https://ajethno.com/2699-chapter-39-oxygenation/ https://www.easynotecards.com/print_list/67193

A client age 80 years, who takes diuretics for management of hypertension, informs the nurse that she takes laxatives daily to promote bowel movements. The nurse assesses the client for possible symptoms of:

hypokalemia.

Mr. Jones is admitted to the nurse's unit from the emergency department with a diagnosis of hypocalcemia. His laboratory results show a serum calcium level of 8.2 mg/dL (2.05 mmol/L). For what assessment findings will the nurse be looking?

muscle cramping and tetany (muscle spasms) hypercalcemia = nausea, vomiting, constipation, bone pain, excessive urination, thirst, confusion, lethargy, and slurred speech. hyperchloremia = diminished cognitive ability and hypertension Constipation is a sign of hypercalcemia.

A nurse is providing care to a client with an extracellular fluid (ECF) volume deficit. The nurse suspects that the deficit involves a decrease in vascular volume based on which finding? Select all that apply.

orthostatic hypotension decreased urine output slow-filling peripheral veins The signs and symptoms of decreased interstitial volume include dry mucous membranes and poor skin turgor.

A client returns to the telemetry unit after an operative procedure. Which diagnostic test will the nurse perform to monitor the effectiveness of the oxygen therapy ordered for the client? pulse oximetry thoracentesis spirometry peak expiratory flow rate

pulse oximetry Explanation: Pulse oximetry is useful for monitoring clients receiving oxygen therapy, titrating oxygen therapy, monitoring those at risk for hypoxia, and postoperative clients. Peak expiratory flow rate is used to monitor severe respiratory diseases and the degree of disease control. Spirometry is used in the postoperative period to measure the volume of air in liters exhaled or inhaled. Spirometry also evaluates lung function and airway obstruction but does not specifically monitor the effectiveness of oxygen therapy. Thoracentesis is a procedure that allows the physician to aspirate pleural fluid for diagnostic or therapeutic purposes.

The nurse writes a nursing diagnosis of "Fluid Volume: Excess." for a client. What risk factor would the nurse assess in this client?

renal failure Excess fluid volume may result from increased fluid intake or from decreased excretion, such as occurs with progressive renal disease. Excessive use of laxatives, diaphoresis, and increased cardiac output may lead to a fluid volume deficit.

A client has been admitted with fluid volume deficit. Which assessment data would the nurse anticipate? (Select all that apply.)

respiratory muscle weakness confusion ventricular dysrhythmia Fluid volume deficit causes a low BP (100/48 mm Hg), poor skin turgor, and an elevated heart rate (128/bpm). Fluid excess can cause crackles and distended neck veins.

The passageways of the kidney permit the urine to flow to the bladder and:

selectively reabsorb or secrete substance to maintain fluids and electrolytes.

The primary extracellular electrolytes are:

sodium, chloride, and bicarbonate.


Conjuntos de estudio relacionados

Environmental Science - Chapter 10

View Set

ITS 121 Quiz 2 Study Guide for Chapter 5

View Set

Rn Concept-Based Assessment Level 2 Online Practice B

View Set